Women health retake

Réussis tes devoirs et examens dès maintenant avec Quizwiz!

A 23-year-old woman presents to the emergency department with right-sided abdominal pain, nausea, and vomiting that began suddenly about 30 minutes ago after she went to a group exercise class. She was recently diagnosed with polycystic ovarian disease. Upon physical examination, she does not have any tenderness to palpation to her abdomen or pelvis and there are no palpable masses or evidence of distension. A pregnancy test is negative. Which of the following is the best way to definitively diagnose her condition?

Direct visualization at the time of surgical evaluation dx:Ovarian torsion

A 24-year-old woman presents to the emergency department with sudden onset of right-sided pelvic pain associated with nausea. Vital signs are T of 97.8°F, HR of 85 bpm, BP of 132/84 mm Hg, and RR of 20/min and have been stable during the patient's 4 hours in the emergency department. Physical exam reveals no vaginal bleeding and right-sided adnexal tenderness during bimanual exam. Pregnancy test is negative. Hemoglobin is 12.5 g/dL. Pelvic ultrasound reveals a right-sided 4 cm ovarian cyst with mild to moderate surrounding blood in the pelvis. The patient's pain is improved following intravenous ketorolac. Which of the following is the best management?

Discharge with pain control dx:Ovarian cysts evaluating the significance of an ovarian cyst are the presence of symptoms, the size of the cyst, whether the cyst is hemorrhagic, and the overall suspicion for malignancy. Ovarian cysts are usually asymptomatic. However, cysts may rupture and cause a sudden onset of severe unilateral lower abdominal pain. Cyst rupture often occurs during sexual intercourse or strenuous activity. It is important to monitor the hemodynamic status (heart rate and blood pressure) in patients with a ruptured ovarian cyst. A) ruled out. A pregnancy test B)The hemoglobin level should be assessed in patients with a suspected ruptured cyst but may be falsely normal initially since patients hemorrhaging lose whole blood. The hemoglobin does not drop until the volume loss of the blood is replenished. C). Suspicion for malignancy is higher in postmenopausal women. best imaging =pelvic ultrasound. TREATMENTS======> Hemodynamic status is assessed by symptoms of hypovolemia (lightheadedness, syncope, orthostasis), signs of hemodynamic instability (tachycardia and hypotension), and a drop in hemoglobin. 1). Patients with low concern for malignancy and hemodynamic stability are considered to have uncomplicated ovarian cyst rupture ===== hence managed outpatient with observation + PAIN MANAGENT 2). pain medication, such as nonsteroidal anti-inflammatory drugs. 3). Symptoms related to uncomplicated ovarian cyst rupture usually resolve within a few days. Patients who are hemodynamically stable but have a large hemorrhage on ultrasound may need to be admitted for observation. 4) hemodynamically unstable at any point may need surgery to stop the bleeding. Furthermore, patients admitted for observation with ongoing bleeding also require surgery.

A 28-year-old G2P1 woman at 36 weeks gestation presents to the ED with uterine contractions, abdominal pain, and vaginal bleeding for the past 2 hours. Her blood pressure is 88/60 mm Hg, and her heart rate is 115 bpm. Physical examination is notable for a rigid, tender uterus. Ultrasound is significant for a retroplacental hematoma. Fetal heart rate tracing is nonreassuring. The patient is prepped for an emergent cesarean section. Which of the following is the most common potential complication of the patient's condition?

Disseminated intravascular coagulation dx:Placental abruption ------------------------------------------------- Retroplacental hematoma is a classic ultrasound finding. A placental abruption of > 50% significantly increases the risk for acute disseminated intravascular coagulation (DIC) and fetal death. Other complications include excessive blood loss resulting in hypovolemic shock, kidney failure, respiratory distress syndrome, multiorgan failure, and death. Management is determined by gestational age, maternal stability, and fetal stability. Emergency cesarean section or peripartum hysterectomy may be needed in life-threatening cases. Placental Abruption Patient will be in third trimester History of hypertension, trauma, or cocaine use Painful vaginal bleeding Labs will show hypofibrinogenemia Tx: fetal monitoring, hemodynamic stabilization, delivery

25-year-old woman who is RhD negative presents to the clinic to establish prenatal care. She is 17 weeks pregnant with normal vital signs and positive fetal heart tones. Prior obstetric history is positive for three preterm births, each progressively earlier in gestation, with no living children. The father of her child is RhD positive (homozygous). A serum maternal anti-D titer is 1:32. Which of the following represents the best next step in the prenatal care of the patient and her fetus?

Doppler velocimetry of the fetal middle cerebral artery Consequences of this include fetal anemia, hydrops fetalis, preterm labor, and fetal demise. Subsequent pregnancies involving RhD-positive fetuses will produce more rapid and aggressive antibody response, with symptoms of fetal demise at an earliergestational age. Prevention of alloimmunization in cases of RhD incompatibility can be accomplished through administration of anti-D immune globulin at 28 weeks of the first and each subsequent gestation. However, patients without access to adequate prenatal care may present during a subsequent pregnancy, when prevention is no longer an option. If RhD incompatibility is suspected due to parental genetics, maternal anti-D titers should be measured serially until a critical titer level (usually 1:16 or 1:32) is reached, at which time, Doppler velocimetry of the middle cerebral artery of the fetus should be measured. Increased velocity through the middle cerebral artery correlates with decreased hemoglobin.

A 24-year-old woman presents with fever, chills, and painful lumps to the groin area for the past three weeks. She is sexually active with men only. Pregnancy test is negative. Upon physical examination, you note tender inguinal and femoral lymphadenopathy bilaterally with some anal fissures. There are no lesions noted on labia majora or minora. Which of the following is the treatment of choice for this patient?

Doxycycline dx:Lymphogranuloma venereum second stage involves painful inguinal lymphadenopathy (appearance of "grooves sign"

If an asymptomatic woman with an IUD is found to have actinomyces incidentally on Pap smear, what is the recommended treatment?

Answer: Leave IUD in place, do not perform a culture and recommend follow up if she becomes symptomatic for pelvic inflammatory disease.

Which medication can be offered to women with risk factors for preeclampsia to prevent preeclampsia?

Aspirin.

pregnant 26-year-old woman presents for an ultrasound at 16 weeks gestation. She has a past medical history significant for a pregnancy loss at 22 weeks gestation prior to her current pregnancy. Ultrasound confirms a shortened cervix. Following cervical cerclage, what additional lifestyle modifications should be recommended for this patient?

Avoid coitus is the only restriction recommended for pregnant women who undergo cerclage. Diagnosis of cervical insufficiency is made through physical exam, ultrasound, or obstetrical history.

A 25-year-old woman presents to the emergency room two weeks postpartum complaining of fevers and breast tenderness. On exam, she is febrile to 102.1°F (39°C) with right-sided breast tenderness and a 4 cm x 4 cm fluctuant mass next to her nipple. What is the best next step in diagnosis?

Breast ultrasound

A 25-year-old G1P0 woman at 11 weeks gestation presents to her obstetrician for routine prenatal care. She has questions about how much calcium she should be taking. She has no significant medical history. She has annual blood work drawn by her primary care provider and has not had any abnormal results. Which of the following is the daily recommendation of calcium for this patient?

1,000 mg ========================== weight [BMI of 18.5-24.9 kg/m2 should gain 25-35 lbs over the course of the entire pregnancy. Patients with a prepregnancy BMI < 18.5 kg/m2 should gain 28-40 lbs during pregnancy, patients with a prepregnancy BMI of 25.0-29.9 kg/m2 should gain 15-25 lbs during pregnancy, and patients with a prepregnancy BMI ≥ 30.0 kg/m2should gain 11-20 lbs during pregnancy.] caloric need of the mother increases by approximately 340 kcal/day in the second trimester and 450 kcal/day in the third trimeste protein intake of 1.1 g/kg/day is recommended, up from 0.8 g/kg/day for nonpregnant women. A carbohydrate intake of 175 g/day is recommended, up from 130 g/day for nonpregnant women. Fiber intake should be around 28 g/day with plenty of fluid intake to reduce constipation. iron 27 mg, calcium 1,000 mg, vitamin D 600 IU, folate 600 mcg, and iodine 220 mcg Women who have had a previous neural tube defect-affected pregnancy should consume 4,000 mcg of folic acid daily. Iron supplementation is important for correcting or preventing iron deficiency anemia. Women with a first- or third-trimester hemoglobin < 11 g/dL, a second-trimester hemoglobin < 10.4 g/dL, or a serum ferritin < 40 ng/mL should receive an additional iron supplement of 30-120 mg/day until the anemia is corrected. Adequate calcium intake may reduce the risk of developing a hypertensive disorderduring pregnancy. Adequate vitamin D levels are associated with a lower risk for a small for gestational age infant. Excessive intake of which micronutrient during pregnancy is associated with fetal goiter? Answer: Iodine.

A 12-year-old girl presents with concerns about not having started menstruating. She states that all her friends have, and she is afraid something is wrong with her. On physical exam, you note breast development is appropriate for her age. In the absence of menses, at what age would the patient require an evaluation for primary amenorrhea?

15 Failure of the menses to appear by age 15 with normal growth and secondary sex characteristics or by age 13 in the absence of secondary sex characteristics, such as breast development, is defined as primary amenorrhea and should prompt further evaluation. Etiologies include hypothalamic-pituitary causes, ovarian causes, hyperandrogenism, disorders of sexual development, pregnancy, and uterine causes. Gonadal dysgenesis is the most common cause of primary amenorrhea. The initial evaluation includes a thorough physical exam and history. A pelvic exam should be performed to assess for hymen patency and the presence of a uterus. MRI of the hypothalamus should be performed when patients have a low or normal FSH and LH, especially if they have high prolactin levels. Patients with a normal uterus, high FSH, and without the features of Turner syndrome should have a karyotype to evaluate for X chromosome mosaicism. Treatment depends on the underlying cause, and hormone replacement therapy is used for patients with permanent hypogonadism.v

A 32-year-old woman who is 37 weeks gestation presents to the maternity ward with a fever of 102.1°F and a heart rate of 115 bpm. She states that she has felt some leaking fluid from her vaginal area for the past 2 days but was unsure of what it was. A pelvic exam reveals fluid leakage from the cervical os. A fetal ultrasound reveals baseline fetal tachycardia. Which of the following antibiotic regimens is the treatment of choice for this patient?

Ampicillin and gentamicin dx: chorioamnionitis serious complication of pregnancy caused by infection or inflammation of the fetal amnion membrane and chorion membrane When should antibiotics be discontinued in the mother? Answer: When she has been afebrile and asymptomatic for 24 hours.

A 32-year-old G3P2 woman presents to the office complaining of heavy menstrual bleeding. Menarche occurred at 12 years of age, and she reports a history of regular menses every 28 days since. After the birth of her second child, her menses were regular, but she now reports heavy menstrual bleeding. She is able to fill a super maxi pad every 2 hours. Her last Pap smear was performed 6 months ago and was normal. Transvaginal ultrasound reveals an 8 cm uterus with a 6 mm endometrial stripe. There are no uterine fibroids. Ovaries are normal and there are no adnexal masses. She is interested in a low-maintenance treatment that will prevent heavy bleeding or one that would cause menses to cease altogether. She is not interested in endometrial ablation or hysterectomy at this time since she is unsure whether she desires to preserve childbearing potential for the future. Which of the following clinical therapeutics is the most appropriate option?

52 mg levonorgestrel-releasing intrauterine device is effective for up to 5 years . It is the only device approved by the Food and Drug Administration to treat menorrhagia (heavy menstrual bleeding). It may also reduce dysmenorrhea. Intrauterine devices prevent pregnancy by a number of methods. For the devices that release levonorgestrel, this medication thickens cervical mucus, preventing sperm from reaching the egg ////////////////////////////////////////////////// 13.5 mg levonorgestrel-releasing intrauterine device (A) that is effective for up to 3 years. It is the smallest intrauterine device available (along with the 19.5 mg device) and has a smaller-diameter inserter. This makes it an available option for nulliparous women. Since it has a lower dosage of levonorgestrel, most patients will continue to have menstrual bleeding with this device. There is a 28 x 30 mm T-shaped 19.5 mg levonorgestrel-releasing intrauterine device (B) effective for up to 5 years. Patients report lighter menses but rarely amenorrhea. The copper intrauterine device (D) is a nonmedicated, 32 x 36 mm T-shaped copper device effective for up to 10 years. It works as a contraceptive by disrupting sperm motility, preventing egg fertilization, and inhibiting fertilized egg implantation. Unlike the other intrauterine devices, menses may be heavier and longer with this device. It is the only device appropriate for emergency contraception.

A 24-year-old woman presents to the emergency room with right-sided pelvic pain and vaginal bleeding. Vital signs are T of 98.4°F, HR of 108 bpm, BP of 124/78 mm Hg, RR of 20/min, and oxygen saturation of 99% on room air. The patient has right-sided adnexal tenderness on physical exam with blood in the vaginal vault. The human chorionic gonadotropin level is 6,000 mIU/mL. Her transvaginal ultrasound is shown above. Which of the following gestational ages is the most common time for this condition to present?

6-8 weeks dx:Ectopic pregnancy

True or false: cigarette smoking increases the risk of preeclampsia.

Answer: False. Cigarette smoking is associated with a lower risk of preeclampsia.

True or false: pregnant women with a history of genital herpes and prodromal symptoms but no active lesions at the time of labor should deliver vaginally.

Answer: False. Prodromal symptoms are an indication for cesarean delivery.

What are some of the teratogenic effects of angiotensin-converting enzyme inhibitors?

Answer: Fetal kidney hypoperfusion, fetal hypotension, fetal growth restriction, and fetal demise.

A 36-year-old G1P0 woman presents to the clinic for a prenatal evaluation. Recently, she has had severe nausea and vomiting. Physical exam reveals a larger than expected uterine size for gestational age, with bilateral adnexal masses noted on bimanual exam. Ultrasound imaging of the uterus is shown above. When is the earliest this condition can be diagnosed?

8 weeks gestation dx:Gestational trophoblastic disease What is considered an acceptable decrease in human chorionic gonadotropin levels after treatment for gestational trophoblastic disease? Answer: A decrease > 10% demonstrated by four values taken weekly for 3 consecutive weeks.

What additional studies are recommended in patients with nonreassuring patterns on fetal heart tracings?

Answer: Fetal scalp stimulation or fetal scalp pH measurement.

Which medication used to treat cystitis can be administered as a single dose?

Answer: Fosfomycin.

Which person is statistically most likely to be phenotypically Rh(D)-negative?

A 40-year-old woman from the Basque Country northern Spain and southern parts of France near the western end of the Pyrenees. The area is culturally unique to the surrounding areas At what gestational age is Rh(D) immunoglobulin typically administered? Answer: 28 weeks.

Which maternal hormone is suppressed by breastfeeding?

Answer: Gonadotropin-releasing hormone (GnRH).

What is the most commoient?n reason a vaginal culture is performed when vulvovaginal candidiasis is suspected?

Answer: If the patient's symptoms are resistant to azole therapy and Candida glabrata is suspected.

What is the main clinical concern in women with functional hypothalamic amenorrhea?

Answer: Impaired bone accrual during adolescence and low bone density during adulthood.

A 30-year-old woman who just delivered a 9 lb 12 oz baby vaginally after a prolonged labor suddenly develops heavy vaginal bleeding, tachycardia and altered mental status. What finding is most likely to be present to confirm the underlying cause of hemorrhage?

A boggy, enlarged uterus upon palpation of the pelvis What is the most common cause of traumatic postpartum hemorrhage? Answer: Cervical laceration.

Excessive intake of which micronutrient during pregnancy is associated with fetal goiter?

Answer: Iodine.

A 27-year-old woman presents to her primary care physician with concern for postcoital pain and bleeding. She reports no fevers or abdominal pain. She is having normal periods. She is sexually active with men only and has had seven lifetime partners, with one new active sexual partner in the last 2 months. They do not always use a condom. On evaluation, a friable cervix is noted with a few shallow ulcerations on her vaginal tissues. She is tender in these areas but has no cervical motion tenderness. There is no active discharge or concerning odor. She otherwise appears well and reports she has never had these symptoms before. Which of the following is the best choice for treatment?

Acyclovir 400 mg oral three times daily dxCervicitis, an acute inflammation of the glandular epithelium of the cervix STI

A 22-year-old woman presents with irregular vaginal bleeding and pelvic pain. Her last menstrual period was six weeks ago and she states that her menstrual cycle is typically 26 to 28 days in length. A urine pregnancy test is positive and a pelvic examination reveals mild right adnexal tenderness. Transvaginal ultrasound shows a right adnexal mass. Serum human chorionic gonadotropin (hCG) level is 3,520 IU/L. Which of the following is the next best step in managing this patient? What are the most common adverse reactions to medication?

Administer methotrexate Answer: Stomatitis or conjunctivitis.

A 28-year-old G3P2002 woman presents to labor and delivery at 33 weeks and 1 day gestational age complaining of high home blood pressure readings. She reports no headache or visual changes. Her serial blood pressures 4 hours apart are 165/115 mm Hg and 173/102 mm Hg. Her urine dipstick shows 3+ protein. Laboratory tests reveal a creatinine of 0.9 mg/dL and a platelet count of 160,000/µL of blood. Which of the following is the best management?

Administration of betamethasone and intravenous labetalol and admission for vaginal delivery dx: Preeclampsia The definitive treatment of preeclampsia is delivery. However, the timing varies based on the gestational age and the presence of severe features. Severe features of preeclampsia include severe hypertension (at least 160/110 mm Hg), severe headache, visual disturbances, kidney dysfunction, hepatic dysfunction, thrombocytopenia, and pulmonary edema. Patients with preeclampsia with severe features should be delivered regardless of gestational age due to the maternal risks. Therefore, the patient in the vignette should be admitted for delivery. Patients with preeclampsia without severe features should be managed expectantly until a gestational age of 37 weeks. What is the first sign of hypermagnesemia in patients being treated with magnesium sulfate to prevent seizures? Answer: Loss of the patellar reflex.

A 28-year-old otherwise healthy pregnant woman presents to the emergency department at 20 weeks gestation with right flank pain. Her other symptoms include dysuria and chills. Vital signs are T of 101.6°F, HR of 114 bpm, BP of 120/80 mm Hg, RR of 20/min, and oxygen saturation of 98% on room air. Physical exam findings include right-sided costovertebral angle tenderness. Urinalysis shows pyuria and bacteriuria. Which of the following is the recommended management?

Admission and initiation of intravenous ceftriaxone dx:Acute pyelonephritis

A 30-year-old nulliparous woman presents with cyclical pelvic pain that has progressively worsened over the last 10 months. She also complains of dysmenorrhea, hematuria, and pain with sexual intercourse. She has been trying to get pregnant but has not been successful. Which of the following physical findings is most suggestive of the suspected diagnosis?

Adnexal masses dx:Endometriosis

A 65-year-old woman presents to the clinic with a painless breast lump. Which of the following is the most important risk factor for the most concerning diagnosis?

Age is the most significant risk factorfor breast cancer. f breast cancer are a hard, immobile, lesion with irregular borders. Breast imaging is used to supplement the physical exam preferred biopsy method is a core-needle biopsy. Screening for breast cancer is recommended to detect cancers early beginning screening at 40 years of age

What is the mechanism of action of leuprolide?

Answer: It is a gonadotropin-releasing hormone analogue. It acts to cause suppression of FSH and LH.

What is the black box warning for metformin?

Answer: Lactic acidosis.

A 24-year-old G2P1 woman at 32 weeks gestation presents to the ED with menstrual-like cramps, low back pain, vaginal pressure, and light vaginal bleeding. She reports no vaginal pain or drug use but smokes two packs of cigarettes each week. Uterine contractions are occurring 5 times every 20 minutes. Her vital signs are T 98.6°F, BP 136/84 mm Hg, HR 88 bpm, RR 20 breaths per minute, and pulse oximetry 98% on room air. Speculum examination reveals cervical dilation of 4 cm without any pooling of fluid into the posterior vaginal fornix. A fetal fibronectin test is positive. Her urinalysis is unremarkable. Fetal heart rate is 151 bpm, and the nonstress test is reactive. Rectovaginal swab for group B streptococcal testing is obtained. Which of the following clinical interventions is the most appropriate at this time?

Ampicillin, betamethasone, and magnesium sulfate dx:Preterm labor Management of patients in preterm labor includes bed rest, oral or intravenous hydration, antenatal corticosteroid therapy (e.g., betamethasone) to enhance lung maturity, group B streptococcal infection prophylaxis (e.g., penicillin, ampicillin) until group B streptococcal testing comes back negative or the patient delivers, antibiotic treatment of any documented urinary tract or sexually transmitted infections, magnesium sulfate for neuroprotection (in patients 24-32 weeks gestation), and transfer to a better-equipped medical facility if necessary. Tocolytic drugs (e.g., indomethacin, nifedipine, magnesium sulfate, terbutaline) may be used to delay delivery for up to 48 hours to allow the antenatal corticosteroid to take full effect.

What kind of findings may a woman at risk for preterm delivery have?

An elevated fetal fibronectin level or short cervical length on ultrasound examination.

A 48-year-old woman presents to the office for her annual physical exam. She has not had a period for over 12 months. Which of the following hormonal changes are likely to occur during the menopausal transition?

An increase in estrone, decrease in estradiol, and no change in testosterone ging follicles become more resistant to gonadotropin stimulation, causing circulating follicular stimulating hormone (FSH) and luteinizing hormone (LH) to increase lead to higher total cholesterol levels, low-density lipoprotein (LDL), and apolipoprotein B levels with the loss of the protective effect of high-density lipoprotein (HDL).

hat are the risk factors for pelvic organ prolapse?

Answer: Increased parity, advancing age, obesity, and increased intra-abdominal pressure.

Which human chorionic gonadotropin level is used as the discriminatory zone for when a gestational sac should be visualized on transvaginal ultrasound?

Answer: 2,000 mIU/mL.

How long after menarche do ovulatory cycles typically start?

Answer: 2-5 years.

In a woman who has a BMI between 18.5 to 24.9 kg/m2, normal weight, what would the recommended weight gain be in a healthy, singleton pregnancy?

Answer: 25 to 35 lbs, 11.5 to 16.0 kg.

At which gestational age in the pregnancy are cervical cerclages typically removed?

Answer: 36-37 weeks gestation or with the onset of preterm labor.

What disorder is associated with preeclampsia that presents prior to 20 weeks of gestation?

Answer: A molar pregnancy.

Which antibiotic should be prescribed for periductal mastitis?

Answer: Amoxicillin-Clavulanate.

What is a theca lutein cyst?

Answer: An ovarian cyst that results from overstimulation by beta-human chorionic gonadotropin, such as occurs during molar pregnancy, multiple gestation, or clomiphene therapy.

When can magnesium sulfate be discontinued?

Answer: At least 24 hours after delivery.

What medication is administered during external cephalic version to relax the uterus?

Answer: Beta-adrenergic agonist, such as terbutaline.

What is the typical fetal response to maternal seizures in eclampsia?

Answer: Bradycardia during and immediately after the seizure.

Which complication of loop electrosurgical excision procedure is characterized by recurrent second-trimester miscarriage?

Answer: Cervical insufficiency.

Which fetal malformation is Zika virus classically associated with during pregnancy?

Answer: Congenital microcephaly.

Which medication can be started during the second trimester in high-risk patients to reduce the risk of developing preeclampsia?

Answer: Low-dose aspirin. Treatment: delivery at 37 weeks (without severe features) and 34 weeks (with severe features) AND prevention of seizures with magnesium sulfate and prevention of permanent maternal organ damage New-onset hypertension < 20 weeks gestation: suspect molar pregnancy

What is the most common site of early hematogenous metastases of a choriocarcinoma?

Answer: Lungs.

What finding on thoracentesis of a patient with a pleural effusion suggests a diagnosis of epithelial ovarian cancer?

Answer: Malignant müllerian cells.

What are some causes of fetal hypoxia? normal FHR range is between 110 and 160 bpm.

Answer: Maternal hypertensive disease, maternal heart disease, maternal infection, placental insufficiency, fetal growth restriction, and umbilical cord prolapse.

Which autosomal dominant condition is characterized by short stature, pulmonary stenosis, hypertelorism, downslanting palpebral fissures with highly arched eyebrows, strikingly blue irises, and a webbed neck?

Answer: Noonan syndrome.

What are risk factors for developing preeclampsia?

Answer: Nulliparity, age greater than 35 years, obesity, diabetes, history of preeclampsia.

Which diagnostic tests are recommended immediately prior to inserting an intrauterine device?

Answer: Pregnancy testing and gonorrhea and chlamydia testing.

In what circumstances can an adolescent undergo a pelvic exam without parental consent? At what age should routine annual pelvic exams begin in nonpregnant women?

Answer: Pregnancy, testing for sexually transmitted diseases, or treating sexually transmitted disease. >21 y.0

What are the characteristics of the sinusoidal pattern in cardiotocography?

Answer: Regular, smooth, and undulating sine wave with a cycle frequency of three to five cycles per minute and an amplitude range of 5 to 15 bpm that persists for at least 20 minutes.

What surgical procedure is most commonly performed in the presence of fallopian tube rupture as a result of an ectopic pregnancy?

Answer: Salpingectomy.

: Why should indomethacin not be used for more than 48 hours in women who are in preterm labor?

Answer: Use for more than 48 hours can cause premature closure of the ductus arteriosus.

When does a fibroadenoma increase your risk of breast cancer?

Answer: When the fibroadenoma is complex, when there is adjacent proliferative disease, or if there is a family history of breast cancer

Deficiency of which essential trace element is associated with impaired taste and smell, night blindness, decreased spermatogenesis, dermatitis, delayed wound healing, and alopecia?

Answer: Zinc.

How are recurrent abortions defined?

Answer: ≥ 2 abortions that occur in sequence in previously healthy pregancies that are < 24 weeks gestation.

A 28-year-old G2P1 Rh-negative woman presents to the clinic for an initial prenatal visit. Which of the following should be performed at this visit?

Anti-D antibody screen What is the quadruple marker test used to screen for? Answer: Spina bifida, Trisomy 21 (Down Syndrome), Trisomy 18 (Edwards Syndrome).

A 22-year-old woman presents to the emergency department with a complaint of acute-onset abdominal pain. She has a low-grade fever, but her vital signs are otherwise normal. On exam, her abdomen is tender, and she has cervical motion tenderness. Purulent vaginal discharge is also noted. She reports being sexually active with multiple casual partners. Her partners do not always use condoms nor does she know much about their medical or sexual histories, but she reports being treated for sexually transmitted infections herself "a few times." A preliminary workup is started for sexually transmitted infections, and a pelvic ultrasound is shown above. Hospital admission is initiated. Which of the following is the best choice for treatment on admission?

Antibiotic therapy alone dx: Pelvic inflammatory disease (PID) can be complicated by a number of processes, including endometriosis, salpingitis, oophoritis, peritonitis, perihepatitis, and tubo-ovarian abscess.

A 32-year-old P0 woman at 33 weeks gestation has confirmed rupture of membranes. The patient is not in labor, and the fetal status is reassuring with a category 1 tracing. The fetus is vertex, and the cervix appears to be closed and long. What is the best next step in the management?

Antibiotics, betamethasone DX: PPROM -------------------------------------------------- Antibiotics,betamethasone, and magnesium (D) would be indicated for fetal neuroprotection at less than 32 weeks gestation. Magnesium is also indicated with the diagnosis of preeclampsia with severe features. Antibiotics, betamethasone, and induction of labor (C) should be performed at 34 weeks or sooner if there are signs of infection or another medical or fetal indication exists.

A 24-year-old G1P1001 woman is preparing for hospital discharge after a spontaneous vaginal delivery with no complications. Which of the following is the most appropriate recommendation to give her regarding when she can return to sexual activity?

As soon as the woman feels ready after a minimum of 2 weeks follow-up postpartum visit 4-6 weeks following delivery is recommended.

A 26-year-old woman presents to your office for health care maintenance. She reports chronic pelvic pain. A review of her records reveals several emergency room visits for acute pelvic pain with a negative evaluation. You are suspicious for domestic violence. She does not report abuse. Her physical examination is normal. Which of the following is the best next step?

Ask about specific types of abuse ---------------------------------------------- The physician should not call the police (B) or discuss concerns with the patient's partner (C) since this represents a major breach in the patient's confidentiality and could place her at risk of social and physical repercussions. It is acceptable to schedule a follow-up visit (D) only after directed questions have been asked and answered. This can enable an open line of communication after the patient leaves the physician's office.

A 65-year-old woman presents to the clinic with right-sided abdominal fullness and early satiety that has developed insidiously over the past 2 months. Physical examination reveals right-sided adnexal fullness. Laboratory testing shows a cancer antigen 125 of 300 units/mL. Transvaginal ultrasound shows a right-sided adnexal mass with solid components and irregular septations. Which of the following is a risk factor for the most likely diagnosis?

BRCA1 genetic mutation dx:Ovarian cancer, postmenopausal women What finding on thoracentesis of a patient with a pleural effusion suggests a diagnosis of epithelial ovarian cancer? Answer: Malignant müllerian cells.

A 24-year-old woman had an uncomplicated vaginal delivery 20 minutes prior to your shift starting. The placenta delivered spontaneously and the reported estimated blood loss was 400 mL. Shortly after receiving sign out, the patient's nurse calls you about bleeding. You present bedside to evaluate. An additional 200 mL of blood is on the peripad, the patient appears pale, and she has ongoing minimal bleeding. Her vital signs are notable for a heart rate of 120 beats per minute and a blood pressure of 95/50 mm Hg. Which of the following characteristics gives this patient a diagnosis of postpartum hemorrhage?

Bleeding associated with signs or symptoms of hypovolemia within 24 hours of delivery dx:postpartum hemorrhage What is secondary (also called late or delayed) postpartum hemorrhage? Answer: Postpartum hemorrhage between 24 hours and 12 weeks after delivery is called secondary, late, or delayed postpartum hemorrhage.

A 25-year-old G1P0 woman presents to her obstetrician for her first prenatal visit. She would like to know what kind of testing will be done at each visit throughout her pregnancy. Which of the following should be obtained at every visit?

Blood pressure

A 55-year-old postmenopausal woman presents to the clinic for her annual physical exam. She has a history of estrogen receptor-positive infiltrating ductal carcinoma and is status post lumpectomy and radiation therapy. Following local treatment, she was started on anastrozole. The patient remains asymptomatic and her most recent mammogram was normal. Which of the following screening evaluations is the most appropriate recommendation for this patient?

Bone density scan It is recommended that a baseline bone density scan (i.e., dual energy X-ray absorptiometry) be performed on postmenopausal women who are taking an aromatase inhibitor, as they are at an increased risk for osteoporosis.

Which of the following cancers would this patient be at significant increased risk of if she is treated with combined menopausal hormone therapy?

Breast

A 28-year-old woman presents to clinic for follow-up of a palpable breast mass. A diagnostic mammogram revealed a suspicious lesion in her right breast, and follow-up ultrasound was concerning for malignancy. During a discussion of surgical options with the patient, she states that she would like further imaging with breast magnetic resonance imaging. Which of the following is an advantage of magnetic resonance imaging in this patient's case?

Breast magnetic resonance imaging is more sensitive than mammogram and ultrasound is also used as a screening method in young women with hereditary breast cancer syndromes who have dense breasts.

A 65-year-old woman presents to her gynecologist after a visit to the ED. The patient reports that she was having abdominal pain last night and went to the ED. Her abdominal pelvic CT scan showed a 10 cm solid, right ovarian mass with multiple small peritoneal masses. Which of the following is most likely to be increased?

Cancer antigen 125

A 65-year-old woman undergoes primary surgical cytoreduction for epithelial ovarian cancer. She has suboptimally cytoreduced disease with greater than 1 cm of residual disease still present. Which of the following is the best pharmaceutical treatment option for this patient?

Carboplatin and paclitaxel intravenously dx: ovarian cancer epithelial ovarian cancer. What are the toxic side effects of paclitaxel? Answer: Neuropathy, myalgias, and weakness. carboplatin administered every three weeks and paclitaxel administered once a week. This regimen is continued for a total of 15 weeks. Ovarian Cancer Patient commonly presents with vague gastrointestinal symptoms, early satiety, bloating, abdominal or pelvic pain Adnexal mass Most common histologic type is epithelial carcinomaTumor marker: CA 125 Rule out germ cell tumors in patients < 30 years old with tumor markers such as hCG and AFP The most common cause of gynecologic death Routine screening not recommended (lack of benefit)

A 19-year-old sexually active woman presents with lower abdominal pain and purulent vaginal discharge of two weeks duration. Temperature is 98.6 °F, heart rate is 70 beats per minute, and blood pressure is 122/74 mm Hg. On exam, the patient expresses tenderness to palpation in right and left lower abdominal quadrants, has purulent cervical discharge, and expresses extreme tenderness with uterine cervical movement. Urine pregnancy test is negative. Which of the following options is the most appropriate therapy?

Ceftriaxone 500 mg IM + doxycycline 100 mg PO twice daily for 14 days + metronidazole 500 mg twice daily for 14 days outpatient dx:Pelvic inflammatory disease (PID) inpatient = cefotetan or cefoxitine plus doxycycline

A 24-year-old woman presents for her annual checkup. She weighs 290 lb after losing 20 lb since starting a new exercise routine. She was recently informed by one of her sexual partners that he tested positive for gonorrhea. What treatment regimen would you offer to her?

Ceftriaxone 500 mg IM once with doxycycline 100 mg bid PO for 7 days

A 26-year-old G4P0121 woman presents to the emergency department at 15 weeks gestation with painless vaginal spotting. She reports she has had two miscarriages during the second trimester that presented similarly. Transvaginal ultrasound confirms cervical dilation and reveals a cervical length of 20 mm. Which of the following is the recommended treatment?

Cervical cerclage dxCervical insufficiency A} cervical cerclage is recommended beginning at 12-14 weeks of pregnancy for women who meet the obstetric history criteria for a diagnosis of cervical insufficiency as listed above[Women who develop a shortened cervix (< 25 mm) ],two consecutive second-trimester pregnancy losses or extremely early preterm births (prior to 28 weeks) associated with minimal or mild symptoms. These women are also often treated with progesterone supplementation beginning at 16 weeks gestation. B}. Women with suspected cervical insufficiency who do not meet the obstetric history criteria should be monitored with frequent surveillance via transvaginal ultrasound. This monitoring should be started at 14-16 weeks gestation. . The diagnosis of cervical insufficiency is limited to singleton gestation pregnancies because second-trimester pregnancy loss in multiple gestation pregnancies is not related to cervical insufficiency.

A 28-year-old G2P1 woman presents to the clinic at 32 weeks gestation for a routine prenatal visit. She has an ultrasound performed to monitor a previously diagnosed placental condition, and it shows persistent homogeneous placental tissue extending over the internal cervical os. The patient reports no vaginal bleeding during her pregnancy.Which of the following is the most appropriate recommendation regarding delivery, assuming the underlying condition persists and the patient remains asymptomatic?

Cesarean delivery at 37 weeks dx:placent previa delivery is recommended between 36-38 weeks gestation What is placenta accreta? Answer: Placental attachment to the myometrium rather than the decidua, which causes the placenta not to spontaneously separate at delivery.

A 26-year-old G4P3 woman presents at 40 weeks gestation for induction of labor. She has a history of type 2 diabetes mellitus, and her body mass index is 37 kg/m2. During the second stage of labor, the fetal head delivers then retracts into the perineum. Gentle downward traction applied to the fetal head, while the patient's legs are elevated and her thighs brought to her abdomen, results in tearing of the perineum but is unsuccessful in delivering the infant. Maneuvers to deliver the posterior shoulder and placing the patient on her hands and knees are also unsuccessful. The obstetrician places the fetal head back into the pelvis. What is the best next step in the management of this patient? AApplying more downward traction with simultaneous fundal pressure BCesarean section CClamping and cutting the umbilical cord DEpisiotomy

Cesarean section (indications on pic in the left ) dx: shoulder dystocia Elevating the legs and flexing the thighs against the abdomen (McRoberts maneuver), placing the patient on all fours (Gaskin maneuver), and maneuvers to deliver the posterior shoulder (Woods screw and Rubin maneuvers) have failed to deliver the fetus. Placement of the fetal head back into the pelvis (Zavanelli maneuver) is done in anticipation of a cesarean section. The 4 most common indications for a C-section delivery are shoulder dystocia, abnormal fetal heart rate tracing, fetal malpresentation, and multiple gestation. ------------------------------------------------------ Episiotomy (D) should not be done if the perineum is already torn during delivery because of the increased risk of extension of the incision, resulting in third- and fourth-degree tears. If the perineum is not already torn, episiotomy may facilitate delivery of the posterior shoulder but does not aid in release of the anterior shoulder. Applying more downward traction with simultaneous fundal pressure (A) is incorrect because it may result in injury to the brachial plexus, further shoulder impaction, or uterine rupture.

A 24-year-old G2P2 woman in the labor and delivery unit develops fever, lower abdominal pain, and foul-smelling, purulent lochia 3 days after a cesarean section for prolonged rupture of membranes. Her vital signs are T of 38.4°C (101.1°F), BP of 130/70 mm Hg, HR of 105 bpm, RR of 18 breaths per minute, and oxygen saturation of 98% on room air. Her pregnancy was complicated by bacterial vaginosis and heavy vaginal colonization with Escherichia coli. Which of the following is the most significant risk factor for the patient's current condition? tx?

Cesarean section is the most important risk factor for postpartum endometritis. Empiric treatment with broad-spectrum antibiotics such as clindamycin and gentamicin has cure rates > 90%. GBS colonized: add ampicillin or use ampicillin-sulbactam Prevention of endometritis includes single-dose prophylactic antibiotic therapy within 60 minutes of skin incision.

Which organism causes lymphogranuloma venereum?

Chlamydia trachomatis.

A 25-year-old woman presents to the clinic for follow-up after testing was performed to evaluate her persistent vaginal bleeding and hemoptysis. She had a vaginal delivery of a term intrauterine pregnancy 4 months ago. Her hCG level is 100,000 mIU/mL. Transvaginal ultrasound shows an enlarged uterus without a fetal heartbeat. Chest X-ray shows multiple pulmonary nodules. Subsequent biopsy of the pulmonary nodules shows sheets of trophoblastic tissue consisting of syncytiotrophoblasts and cytotrophoblasts without villi. Which of the following is the most likely diagnosis?

Choriocarcinoma Molar pregnancies are often suspected by the snowstorm appearance due to swollen villi seen on transvaginal ultrasound, but the definitive diagnosis requires a biopsy Why do hyperthyroidism and theca lutein cysts occur during gestational trophoblastic disease? Answer: The alpha subunit of human chorionic gonadotropin closely mimics thyroid-stimulating hormone and luteinizing hormone.

A 23-year-old woman presents to her gynecologist due to fever, nausea, vomiting, and lower abdominal pain. She is currently sexually active and uses condoms occasionally. Vital signs are T of 38.4°C (101.1°F), BP of 138/78 mm Hg, HR of 90 bpm, RR of 18 breaths per minute, and oxygen saturation of 99% on room air. Physical examination is significant for bilateral lower abdominal tenderness to palpation. Pelvic examination is significant for purulent endocervical discharge, cervical motion tenderness, and uterine tenderness without adnexal masses. The patient is sent to the ED and admitted for treatment. Which of the following is a complication of the most likely diagnosis?

Chronic pelvic pain dx: PID Pelvic Inflammatory Disease (PID) History of multiple sexual partners or unprotected intercourse Lower abdominal pain, cervical motion tenderness (Chandelier sign), painful sexual intercourse PE will show mucopurulent cervical discharge Most commonly caused by Chlamydia trachomatis Outpatient treatment is ceftriaxone + doxycycline + metronidazole Fitz-Hugh-Curtis syndrome: perihepatitis + PID

A 32-year-old woman who is 24-hours post-op after delivering her first child via C-section suddenly develops abdominal pain, foul-smelling vaginal discharge, and uterine tenderness. Her vitals are O2 saturation at 99% on room air, blood pressure at 132/86 mm Hg, heart rate at 115 beats per minute, and temperature 102°F. What is the appropriate treatment for this patient's suspected diagnosis?

Clindamycin and gentamicin dx:endometritis, Postpartum Endometritis Patient will be postpartum, early-onset disease < 48 hours after delivery (C-section more common) Fever, abdominal pain, foul-smelling lochia PE will show uterine tenderness Labs will show leukocytosis Most common postpartum infection Treatment is clindamycin + gentamicinGBS colonized: add ampicillin or use ampicillin-sulbactam

A 28-year-old woman presents to the clinic due to infertility. She has infrequent menstrual cycles and has been unable to become pregnant despite 12 months of frequent unprotected intercourse. Physical examination is unremarkable other than hirsutism and inflammatory acne. Her husband has been evaluated and had a normal semen analysis. Which of the following is the recommended treatment for infertility in this patient?

Clomiphene citrate What are common adverse effects of clomiphene citrate? Answer: Hot flashes, abdominal distention and pain, nausea and vomiting, and breast discomfort. dx :polycystic ovary syndrome (PCOS)

A 36-year-old G2P1 woman presents for a routine obstetrics visit. She feels well and has no complaints. She reports occasional instances of heartburn and has to wear compression stockings daily while at work to prevent lower extremity swelling and discomfort. She feels occasional pelvic pressure and has been urinating more frequently. She reports no burning with urination, flank pain, or hematuria. Yesterday, she felt a gush of fluid vaginally and had to rush to the bathroom because she was worried about incontinence. Since then, she has been leaking a "very small amount of clear-yellow urine." What result of nitrazine testing will confirm her diagnosis

Color change to blue dx:prelabor rupture of membranes Amniotic fluid has a pH of 7.0-7.3, which will cause a color change to blue. Normal vaginal pH is 3.8-4.2 and the pH of urine is usually < 6.0. If the nitrazine paper is yellow or green,

You are reviewing results from a 31-year-old woman who underwent recent cervical cancer screening. Her cervical cytology shows atypical squamous cells of undetermined significance, and her human papillomavirus test is positive. Her pelvic exam does not reveal any gross cervical lesions. Which of the following is the recommended next step for her?

Colposcopy with endocervical sampling atypical squamous cells of undetermined significance (ASC-US) cervical intraepithelial neoplasia grade 1 (CIN 1), also called low-grade squamous intraepithelial lesion (LSIL), and cervical intraepithelial neoplasia grade 3 (CIN 3), also referred to as high-grade squamous intraepithelial lesion (HSIL).

A 21-year-old woman presents to the office with a bump on her labia that appeared 3 days ago. It is not painful to touch nor pruritic. She had unprotected intercourse with a man about 3 weeks ago and has not had any sexual contact with anyone else since. On physical exam, there is a 2 cm nontender, ulcerated, round, skin-colored lesion with an elevated rim that is located on the inside of the right labium majus. There are also palpable lymph nodes to the inguinal area bilaterally. Which of the following tests will provide a definitive diagnosis of her condition?

Darkfield microscopy dx:Syphilis caused by Treponema pallidum, Primary syphilis is the earliest stage, characterized by the presence of a highly infectious painless chancre to the genitals. They are solitary, erythematous, raised, and firm to touch and will ulcerate over time, creating a surrounding crater with slightly elevated edges. These lesions usually heal by themselves within 3-6 weeks. There may also be associated regional lymphadenopathy. If the disease is untreated, secondary syphilis will develop about 4-10 weeks after the chancre appears. This stage occurs due to the reproduction of the spirochete and systemic invasion causing a variety of generalized symptoms such as malaise, fever, muscle pain, joint pain, lymph node enlargement, and rash. The rash is nonpruritic and generalized and described as discrete, reddish-brown macules that typically involve the palms and soles. They can also be found on the mucous membranes. Patients may also have condylomata lata (highly infectious grayish-white lesions found in moist areas) and moth-eaten alopecia latent phase (but will remain seroreactive), while about one-third of patients will develop tertiary syphilis. Latency may last from a few years up to 25 years before the next stage develops. Tertiary syphilis is slowly progressive and mainly affects the cardiovascular system and central nervous system, but it can affect any organ throughout the body. Cardiovascular syphilisdevelops about 10 years after the primary infection, causing aneurysm formation of the aorta. Neurosyphilis can cause damage to the brain and spinal cord at any stage of the disease causing gait abnormalities, disruption in pain and temperature sensations, memory and speech impairment, psychosis, and progressive dementia. dx:nontreponemal and treponemal. Either can be used for screening, but confirmatory testing is necessary for both. Primary: painless chancre Secondary: lymphadenopathy, condyloma lata, rash on palms and soles Tertiary: gummas VDRL and RPR positive 1-4 weeks after infection Primary or secondary: IM benzathine penicillin G, 1 dose Tertiary: IM benzathine penicillin G qwk for 3 weeks

A 28-year-old woman with a past medical history of diabetes mellitus presents to the clinic with irregular menstrual cycles. She says her cycles occur every 2 months. Physical examination reveals male-pattern facial hair and inflammatory acne. Which of the following is the recommended treatment for her hyperandrogenic symptoms?

Combined oral contraceptive pill dx:Polycystic ovary syndrome (PCOS) Hyperandrogenism causes anovulation, which can lead to an accumulation of cysts within the ovaries. The classic features of PCOS include irregular menses (amenorrhea or oligomenorrhea) or abnormal uterine bleeding (related to ovulatory dysfunction), infertility, endometrial hyperplasia and increased risk of endometrial cancer, type 2 diabetes mellitus, and metabolic syndrome. Treatment is combination oral contraceptive pills, lifestyle changes, metformin Most common cause of infertility ----------------------------------------------------- Clomiphene (A) is an ovulation-inducing medication used to treat infertility in patients with PCOS. Spironolactone (D) is an antiandrogen medication used as an adjunct to treat the hyperandrogenic symptoms caused by PCOS. Spironolactone is recommended once patients have not had adequate improvement with 6 months of combined oral contraceptives. The first-line treatment for the patient in the vignette should be combined oral contraceptives. Levonorgestrel-releasing intrauterine device (C) is a progesterone-only hormonal contraceptive. These are not recommended for the treatment of hyperandrogenic symptoms

A 28-year-old otherwise healthy woman presents to the gynecology clinic with 24 hours of right-sided breast pain, redness, and swelling. She is 5 weeks postpartum. Vital signs are T of 100.6°F, HR of 98 bpm, BP of 120/80 mm Hg, RR of 20/min, and oxygen saturation of 98% on room air. Breast examination reveals a local area of erythema and firm edema on the right breast. Which of the following is the recommended treatment?

Continuation of breastfeeding and dicloxacillin cool compress dx : lactational mastitis

A 25-year-old G1P0 woman at 28 weeks gestation presents to her obstetrician for a routine visit. Her blood pressure was previously normal, until her 24-week visit when it was measured at 142/92 mm Hg. Today, her blood pressure is 144/92 mm Hg. She reports no vision changes, headache, or abdominal pain. Urinalysis is significant for 2 protein. Complete blood count and kidney function testing is unremarkable. Which of the following is the most appropriate treatment at this time?

Continued monitoring dx: preeclampsia Patients with preeclampsia without severe features should be managed expectantly until a gestational age of 37 weeks. True or false: cigarette smoking increases the risk of preeclampsia. Answer: False. Cigarette smoking is associated with a lower risk of preeclampsia.

A 27-year-old woman presents to her gynecologist with a left-sided breast mass. Physical examination reveals the presence of a 2 cm, firm, immovable, single lesion on the left breast at the 11 o'clock position. An ultrasound reveals a well-visualized, hypoechogenic 2 cm tall x 1.5 cm wide irregular lesion. Which of the following is the most appropriate diagnostic study at this time?

Core-needle biopsy

A 25-year-old woman presents for her initial prenatal visit. What is the most accurate method for determining her estimated date of delivery?

Crown-rump length

A 27-year-old woman who is positive for HIV presents to the clinic with bumps near her anus. On exam, there are skin-colored cauliflower lesions on the perianal skin. Which of the following is the best clinician-administered treatment?

Cryotherapy dx Condyloma acuminata (genital warts) is a manifestation of anogenital infection , Human papillomavirus types 6 and 11 are responsible

A 35-year-old woman presents to a women's health clinic requesting oral contraceptive therapy. The physician assistant decides to prescribe a combined estrogen-progestin hormonal contraceptive. Which of the following is a relative contraindication to combined estrogen-progestin oral contraception for this patient? A Body weight > 198 lbs (90 kg) B Current tobacco use of 10 cigarettes/day C History of stroke D Migraine with aura

Current tobacco use of 10 cigarettes/day =========================================== absolute contraindications, to combined estrogen-progestin hormonal contraception: current breast cancer, severe decompensated cirrhosis, acute deep vein thrombosis or pulmonary embolism, high risk for recurrence of deep vein thrombosis or pulmonary embolism, major surgery with prolonged immobilization, migraine with aura,

A previously healthy 29-year-old G2P1A0 at 37 weeks and 4 days gestation presents to obstetric triage complaining of right upper quadrant pain that has been consistent since this morning. She also endorses a mild headache. Her blood pressure is 161/90 mm Hg and then 165/93 mm Hg on repeat 10 minutes later. Her pulse is 80 beats per minute. The fetal heart tracing is reassuring. Her physical exam is notable for hyperreflexia and her laboratory testing is remarkable for platelets at 80,000 platelets/microL. Which of the following is the best next step?

Delivery dx: preeclampsia w/ severe

Which of the following should be addressed as a likely result of episiotomy as compared to a spontaneous vaginal laceration when discussing the possibility of a vaginal laceration with the patient?

Dyspareunia [look at image]

A 32-year-old woman presents to the clinic to discuss recurrent miscarriages. She reports three miscarriages that occurred at 18 weeks, 19 weeks, and 21 weeks. She reports that, when she miscarries, she has painless vaginal bleeding without significant contractions. Which of the following is a risk factor for the suspected diagnosis?

Ehlers-Danlos syndrome dx:cervical insufficiency It occurs most often in the late second trimester. Cervical insufficiency may have congenital or acquired etiologies . The diagnosis of cervical insufficiency is made based on obstetric history, physical exam, ultrasound findings, or a combination of these. It can be diagnosed based on obstetrics history in women with at least two consecutive second-trimester pregnancy losses or early premature births (< 28 weeks gestation) that are associated with relatively painless early cervical dilation or at least three preterm births prior to 34 weeks gestation in which other causes have been excluded.

Which of the following clinical scenarios is the most concerning for the diagnosis of endometrial cancer?

Endometrial Cancer Peak incidence is in postmenopausal patients between age 60-70 years Most common type is adenocarcinoma Risk factors: nulliparity, obesity, unopposed estrogen (postmenopausal estrogen therapy without progestin), tamoxifen Sx: abnormal vaginal bleeding Dx: endometrial biopsy Tx: total hysterectomy AND bilateral salpingo-oophorectomy

A 46-year-old woman presents to your office for a second opinion. She has been experiencing abnormal uterine bleeding for more than two years. Her periods are heavy, prolonged, and associated with clotting. Imaging of her uterus was unremarkable. A recent hysteroscopy with directed biopsies was negative. She has three children, all born by uncomplicated vaginal deliveries, and is not interested in future conception. She declines hormonal therapy and would like to proceed with surgical therapy. Which of the following is recommended?

Endometrial ablation Abdominal hysterectomy (A) is a definitive treatment for AUB but is a costly and morbid procedure and should be kept as a last resort. Furthermore, a less invasive approach to hysterectomy is preferred

A 24-year-old woman presents to the clinic reporting infertility. Physical exam reveals hirsutism, obesity, and a fasting glucose of 210 mg/dL. Pelvic ultrasound reveals multiple simple cysts in each ovary. Which of the following is this patient at increased risk of developing? ABreast cancer BEndometrial cancer COvarian cancer DPancreatic cancer

Endometrial cancer dx:Polycystic ovary syndrome

A 28-year-old nulliparous woman presents to the clinic reporting left lower quadrant pain and severe dysmenorrhea. A pelvic ultrasound reveals a left adnexal complex mass that has smooth walls with homogeneous internal echoes that have a ground-glass appearance. Which of the following ovarian masses is the most likely diagnosis?

Endometrioma dxOvarian masses A patient with an endometrioma may report symptoms of endometriosis, including severe dysmenorrhea, dyspareunia, and abdominal pain that is not localized to the uterus during menses. On physical exam, a patient with an endometrioma may have a palpable adnexal mass and discomfort on palpation. The best modality to distinguish adnexal masses is transvaginal ultrasound. On ultrasound, an endometrioma appears smooth-walled with homogeneous internal echoes that have the appearance of ground-glass. The fluid inside endometriomas is old blood and appears chocolate-colored on biopsy, so these cysts are often referred to as chocolate cysts. Treatment of endometrioma includes observation with serial ultrasounds or surgicalremoval. Surgical removal provides definitive diagnosis, relief of symptoms, and protection against possible ovarian torsion or cyst rupture. However, surgery carries the risk of intraoperative complications, such as hemorrhage or infection, and slightly decreases fertility.

A 68-year-old woman presents to the clinic with progressive lower abdominal bloating for the past six months. Ascites is present on exam, and a right-sided adnexal mass is palpable. Pelvic ultrasonography reveals a solid complex mass on the right ovary and confirms presence of associated ascites. Which of the following is the most likely diagnosis?

Epithelial carcinoma ovarian cancer

7 cardinal movements of decent A 24-year-old G2P1 woman is in the second stage of labor. She is 10 cm dilated and 100% effaced. Fetal station is +3. She reports feeling the urge to push, and she is most comfortable in the supine position. With her next contraction, she pushes for three 10-second intervals. The fetal head is delivered and a nuchal cord is freed. Which of the cardinal movements of labor will be the next to occur?

External rotation

A 54-year-old woman presents due to vaginal discomfort. Symptoms began a few years ago and have become progressively worse. She reports daily discomfort and is no longer interested in sexual intercourse because of the pain. Her medical history includes seasonal allergies, for which she takes daily loratadine. Her past surgical history includes a tonsillectomy 10 years ago, partial hysterectomy 3 years ago, and a cholecystectomy 2 years ago. She reports no hot flashes, insomnia, mood swings, or difficulty with memory. A vaginal exam reveals sparse pubic hair, a narrowed vaginal introitus, atrophic vaginal rugae, and pale vaginal tissue. Which of the following is the most appropriate treatment option for this patient?

Estradiol 0.01% vaginal cream What vaginal pH is associated with vaginal atrophy? Answer: ≥ 5.

A 32-year-old woman presents with complaints of painful lumps in her breasts that worsen with her menstrual cycle. Which of the following hormones is considered a causative factor in the development of the suspected diagnosis?

Estrogen

A 24-year-old woman presents to the gynecology clinic for evaluation of breast tenderness and bloating that occurs the week before menses. She also says she is irritable, and the symptoms together make it difficult for her to work. These symptoms occur consistently month to month. Which of the following changes may help with this condition?

Exercising consistently dx: Premenstrual syndrome (luteal phase) of the menstrual cycle and impair some aspect of the patient's life. Common behavioral symptoms include irritability, labile mood, anxiety, sad or depressed mood, appetite changes or food cravings, and diminished interest in activities. Common physical symptoms include abdominal bloating, fatigue, breast tenderness, hot flashes, and dizziness. The diagnosis of premenstrual syndrome is suggested by these symptoms occurring consistently prior to menses. Thyroid disease should be ruled out by checking thyroid-stimulating hormone level. Patients with suspicion for premenstrual syndrome should record their symptoms for 2 months in relation to their menstrual cycle

A 24-year-old woman presents to clinic for her annual gynecologic exam. She has no concerns and takes no medications. She is sexually active with women only. During her pelvic exam, her physician palpates an irregular uterine contour. Further ultrasound workup reveals one 3 cm intramural fibroid. Which of the following is the most appropriate next step? ACombined oral contraceptive pill BExpectant management CLevonorgestrel intrauterine device DNonsteroidal anti-inflammatory drug

Expectant management In this patient who is asymptomatic and does not desire contraception, . Nonhormonal medications include nonsteroidal anti-inflammatory drugs and antifibrinolytics, which target dysmenorrhea and heavy bleeding, respectively. Hormonal medications include oral contraceptive pills, the levonorgestrel intrauterine device, androgenic steroids, and gonadotropin-releasing hormone (GnRH) agonists, which regulate the menstrual cycle. GnRH agonists such as leuprolide additionally may shrink fibroids but can cause menopausal symptoms.

A 28-year-old woman, G2P1, presents at 38 weeks gestation complaining of painful contractions and mild vaginal bleeding. Her prenatal care course is complicated by cocaine and tobacco use. On physical exam, her uterus is firm. Fetal heart rate tracing reveals a baseline of 135 bpm with moderate variability in the absence of accelerations. Tocometer reveals regular contractions. A vaginal exam performed reveals a cervix that is 7 cm dilated and a fetus in cephalic presentation. A transabdominal ultrasound is suspicious for placental abruption as the etiology of her vaginal bleeding. Which of the following is the most appropriate management?

Expectant management dx:Placental abruption What is the main sign of fetal anemia caused by placental abruption? Answer: Fetal acidosis.

A 30-year-old pregnant woman with a history of uncontrolled asthma presents to the obstetrics clinic at 30 weeks gestation for a routine prenatal visit. Her blood pressure measures 152/108 mm Hg and 154/106 mm Hg several hours later on recheck. She has no history of hypertension, and she reports no chest pain, shortness of breath, headache, vision changes, or abdominal pain. Laboratory studies include a urine dipstick negative for protein, 180,000 platelets/µL, creatinine of 0.7 mg/dL, aspartate aminotransferase of 25 U/L, and alanine aminotransferase of 28 U/L. Which of the following antihypertensives is the most appropriate choice to initiate in this patient?*********

Extended-release nifedipine dxGestational hypertension Which intravenous medications are used to acutely lower hypertension in pregnant patients? Answer: Hydralazine and labetalol.

A 30-year-old pregnant woman with a history of uncontrolled asthma presents to the obstetrics clinic at 30 weeks gestation for a routine prenatal visit. Her blood pressure measures 152/108 mm Hg and 154/106 mm Hg several hours later on recheck. She has no history of hypertension, and she reports no chest pain, shortness of breath, headache, vision changes, or abdominal pain. Laboratory studies include a urine dipstick negative for protein, 180,000 platelets/µL, creatinine of 0.7 mg/dL, aspartate aminotransferase of 25 U/L, and alanine aminotransferase of 28 U/L. Which of the following antihypertensives is the most appropriate choice to initiate in this patient?

Extended-release nifedipine dx:Gestational hypertension Which intravenous medications are used to acutely lower hypertension in pregnant patients? Answer: Hydralazine and labetalol. Patients with blood pressure of ≥ 160 mm Hg systolic or 110 mm Hg diastolic are considered to have severe hypertension and are treated similarly to patients with preeclampsia. Lifestyle interventions are important. Patients should be educated to avoid strength training and generally avoid aerobic exercise unless it is documented that aerobic exercise does not increase the patient's blood pressure. monitor fetal movement daily and to call if it is decreased or absent The most common oral antihypertensives used in pregnancy are labetalol, extended-release nifedipine, and hydralazine. Antenatal corticosteroids, such as betamethasone, should be administered to women who are < 34 weeks pregnant and are felt to be at increased risk of labor within 7 days. The timing of delivery varies according to the severity of hypertension. Women with hypertension consistently > 140 mm Hg systolic or 90 mm Hg diastolic are typically delivered at 37 weeks. Most women with gestational hypertension become normotensive within the first week postpartum.

A 37-year-old G4P1012 woman at 12 weeks gestation presents to the clinic with abdominal pain and vaginal bleeding. The patient states that she had a sudden episode of vaginal bleeding yesterday and has since had abdominal pain with vaginal spotting. Vital signs are HR 74 bpm, BP 120/72 mm Hg, and RR 16 breaths per minute. Pelvic exam reveals opened cervical os with evidence of fetal tissue. What is the most common cause of the suspected diagnosis?

Fetal chromosomal abnormalities

A 28-year-old woman who is G1P0 and 30 weeks pregnant presents to the labor and delivery department reporting pelvic pressure and some mild contractions that vary from 5 to 15 minutes apart. Fetal heart sounds are normal, and there is no pooling of fluid in the vaginal vault. Which of the following tests should be ordered to predict the risk of preterm delivery?

Fetal fibronectin test (look at the diagram on the side ) treatment to prevent preterm labor Progesterone and the cervical cerclage procedure in women with a shortened cervix may also be used to help prevent preterm birth in this population. Women who are between 23 and 34 weeks gestation should receive a short course of corticosteroids (e.g., betamethasone or dexamethasone) to promote fetal lung maturity, which is shown to help decrease the risk of respiratory distress syndrome, intracranial hemorrhage, and death. Tocolytics can help prolong the pregnancy long enough to administer corticosteroids. Magnesium sulfate is commonly used and has neuroprotective effects for the infant. Intravenous terbutaline, oral nifedipine, and oral indomethacin can also be used short-term. Tocolytics are not recommended to be used longer than 48 hours.

A 17-year-old girl presents to the office with concerns about a new breast mass. She reports the mass has been present for the last 4 months and seems to get larger around the time of her period. The mass is tender when it is enlarged, but it usually regresses back to its original size after each menstrual cycle and is no longer tender. Physical exam reveals a well-circumscribed, smooth, rubbery, mobile 1.8 cm mass in the right upper quadrant of the right beast. It is nontender, and there are no overlying skin changes. There is no axillary lymphadenopathy. What is the most likely diagnosis?

Fibroadenoma ------------------------------------------------- A breast cyst (B) is more common in women aged 35-50 and may fluctuate in size with the menstrual cycle. The texture of a breast cyst depends on the type of cyst, but it is commonly fluctuant instead of rubbery.

A 35-year-old woman presents to her gynecologist with complaints of breast pain. The patient reports increased breast pain and swelling for 3 days prior to her menstrual cycle for the past several years. She became concerned when she noticed a mass in her right breast. Physician exam is positive for a small, fixed lesion in the upper outer quadrant of the right breast. Ultrasound shows a 1 cm lesion, and biopsy is recommended. Core needle biopsy reveals tissue nonproliferation with fibrotic changes. What is the most likely diagnosis for this patient?

Fibrocystic disease

A 42-year-old G3P2 woman presents in early labor. She is 39 weeks pregnant. Her first two pregnancies presented cephalically and were delivered vaginally at term without complication. One child was 8 pounds, 12 ounces and the other was 9 pounds, 7 ounces. Apgar scores at 1 and 5 minutes were 8 and 9 for both children. Her current pregnancy has progressed normally. Her glucose screening test performed at 26 weeks gestation was elevated at 151 mg/dL. Her 3-hour glucose test returned within normal limits. She is obese and has gained the appropriate amount of weight during this pregnancy. On physical exam, she is in slight discomfort. Contractions are occurring regularly every 5 minutes. She is 3 cm dilated and 100% effaced. The fetus is in frank breech presentation. Which aspect of the patient's history and physical exam increases the risk of a prolapsed umbilical cord?

Frank breech presentation

A 25-year-old G2P1 woman at 33 weeks gestation presents to the emergency department after feeling a gush of vaginal fluid 2 hours ago. On speculum examination, clear fluid is seen pouring out of the cervical os and pooling into the vaginal vault. Nitrazine testing reveals a pH of 7.2, and microscopic examination demonstrates a ferning pattern. Ultrasound is significant for oligohydramnios. Which of the following is the most common risk factor for the patient's condition?

Gardnerella vaginalis infection dx:preterm prelabor rupture of membranes (PPROM) If this occurs before 37 weeks gestation, it is considered PPROM. What is the recommended route, dosage, and duration of metronidazole prescribed for pregnant patients with bacterial vaginosis? Answer: Metronidazole 500 mg PO bid for 7 days.

A 16-year-old high school gymnast presents to her gynecologist with concerns about menstruation. She states that most of her friends started menstruating in middle school, and she is worried because she has never had a period. Her vital signs are normal, her height is in the 5th percentile, and her BMI is 20.1 kg/m2. She states that she has always been the shortest girl in her class. No abnormal weight changes are noted on her growth chart. The patient has appropriate secondary sexual characteristics, and her genitourinary examination is unremarkable. Physical examination is significant for low-set ears, low hairline, and a high arched palate. Urine pregnancy test is negative. Which of the following is the most common cause of the patient's condition?

Gonadal dysgenesis dx:Primary amenorrhea The most common cause of primary amenorrhea is gonadal dysgenesis. Patients with Turner syndrome will present with distinct physical features, such as short stature, low-set ears, low hairline, high arched palate,webbed neck, and widely spaced nipples Followed by primary ovarian insufficiency (46,XX)

Which class of medications do leuprolide acetate and goserelin acetate belong to?

Gonadotropin-releasing hormone agonists.

A 32-year-old woman who is lactating presents to the office due to localized breast pain and malaise. Relevant findings on physical exam include a temperature of 100.9°F with a fluctuate, tender, erythematous mass in the upper outer breast quadrant. The patient has attempted to treat her symptoms with warm compresses, massage, and a course of antibiotics prescribed by her primary care provider. Which of the following results would a culture of aspirated fluid most likely have?

Gram-positive cocci in cluster dx:breast abscess Which risk factor is associated with recurrent breast abscesses? Answer: Smoking. incision and drainage is recommended for all abscesses despite antibiotic therapy. Patients without risk factors for methicillin-resistant S. aureus (e.g., recent surgery, hemodialysis, injection drug use, HIV infection) should be prescribed dicloxacillin or cephalexin, while those with risk factors require clindamycin or trimethoprim-sulfamethoxazole. Patients who demonstrate hemodynamic instability or other signs of severe infection should be admitted and treated with intravenous vancomycin. Lactating mothers should be encouraged to continue to nurse from the affected breast to promote full drainage of the breast and provide pain relief.

A 19-year-old woman presents with a tender genital ulcer that she noticed 2 days ago. She also reports dysuria and dyspareunia. On exam, you note a genital ulcer that has a ragged border and purulent base. A palpable, tender left inguinal lymph node is also present. Which of the following organisms is the most likely causative organism?

Haemophilus ducreyi a gram-negative, fastidious rod that clumps in long parallel strands producing the so-called "school of fish" or "railroad track" appearance. dx:Chancroid unilateral tender inguinal lymphadenopathy (buboes) that appears 1 to 2 weeks after onset of the ulcer. Diagnosis is often made clinically. Serologic tests (rapid plasma reagin, venereal disease research lab) should be obtained to exclude syphilis. Treatment of chancroid consists of antibiotic therapy and drainage of a fluctuant inguinal lymph node if present. Antibiotic options include azithromycin, ceftriaxone, or erythromycin. Chancroid Risk factors: sexually active Sx: painful genital ulcers PE: papule evolves to a pustule which ulcerates, ulcers on an erythematous base covered by a gray or yellow purulent exudate and painful lymphadenopathy (bubo) Caused by Haemophilus ducreyi Tx: ceftriaxone 250 mg IM or azithromycin 1 g oral

A 35-year-old woman undergoes a transvaginal ultrasound showing hypoechoic, round, and well-circumscribed uterine tumors. Which of the following is the most common symptom of the most likely diagnosis?

Heavy menses dx:Fibroids levonorgestrel-releasing intrauterine device is a common option for women who wish for the least invasive treatment. Fibroids usually become smaller or resolve in postmenopausal women. Hysterectomy is the definitive treatment, and fibroids are the most common indication for hysterectomy.

Which of the following is the most important risk factor for shoulder dystocia? ADiabetic pregnancies with normal fetal birth weight BFemale sex CHigh birth weight in nondiabetic pregnancies DPreterm birth

High birth weight in nondiabetic pregnancies

A 57-year-old woman presents for a follow-up to a recent abnormal breast exam. She was noted to have a nontender, hard, fixed breast lump with associated enlarged axillary lymph nodes on her annual breast exam. Her mammogram was also significant for a spiculated mass highly suspicious for malignancy. What is the most common form of this patient's diagnosis?

Invasive ductal carcinoma

A woman who closely monitors her menstrual cycles notices clear cervical mucus similar to raw egg white. Which of the following describes the estrogen and progesterone hormone levels at her most likely stage of the menstrual cycle?

High estrogen and low progesterone estrogen levels rise, which produces a spike in luteinizing hormone that triggers ovulation. The rising estrogen levels also cause the cervical mucus to become thin, giving sperm the best chance for fertilization. The cervical mucus at this stage in the cycle is sometimes described as similar in consistency to a raw egg white.

A 46-year-old woman presents to her gynecology office to discuss her colposcopy results. She was told the results were significant for carcinoma, and she needs a total abdominal hysterectomy with a bilateral salpingo-oophorectomy. What is the greatest risk factor for this patient's diagnosis?

Human papillomavirus dx:cervical carcinoma

A 36-year-old G2P2 woman with no history of prenatal care for her current pregnancy had a spontaneous home vaginal delivery at 35 weeks gestation and presents with her baby 2 hours after delivery because she thinks he is sick. The infant is admitted to the neonatal intensive care unit with skin edema, ascites, pericardial effusion, pleural effusion, and a hemoglobin of 4.9 g/dL. The mother had no prenatal or postnatal care for her first pregnancy, which occurred 3 years ago. Her first child was delivered vaginally at home without any complications. The mother reports cocaine use during this pregnancy. Which of the following is the most likely contributing factor for the infant's condition?

Hydrops fetalis is a complication of hemolytic disease of the fetus and newborn and is characterized by skin edema, ascites,pericardial effusion, pleural effusion, and severe anemia (e.g., fetal hemoglobin < 5 g/dL or a hematocrit < 15%). Thrombocytopenia and neutropenia may also be present. Rh Isoimmunization Rh-negative mothers exposed to Rh-positive blood → anti-Rh antibodies Subsequent pregnancies: jaundice, anemia, fetal hydrops, fetal death Anti-D globulin at 28 weeks (and within 72 hrs of delivery if infant is Rh+)

Which of the following electrolyte imbalances is associated with Hyperemesis gravidarum Which of the following is most likely to be associated with the patient's condition?

Hypokalemia Starvation ketosis weight loss, dehydration, starvation ketosis, hypochloremic metabolic alkalosis, hypokalemia, and transient liver enzyme elevation. https://roshreview.imgix.net/238936c4f5ac62a85b69e10f3bea110f_Image%20-%20Hyperemesis%20Gravidarum%20%408x.png?w=3654&fit=max&q=30&auto=compress

A 68-year-old woman presents to clinic to discuss surgical management of her pelvic organ prolapse. She has had stage III prolapse of the uterus for the past five years. Pelvic muscle exercises and pessary use have not relieved her symptoms of heaviness and discomfort. Her past medical history is significant for sarcoidosis for which she takes oral prednisone. Which of the following is the most appropriate surgical option for this patient?

Hysterectomy with uterosacral ligament suspension Between which structures does the sacrospinous ligament run? Answer: Sacrum/coccyx and the ischial spine.

A 38-year-old woman presents along with her 42-year-old husband to discuss difficulties conceiving. They have had consistent, unprotected intercourse for 8 months without a positive pregnancy test. She has never been pregnant, and he has never fathered a child. She has a history of regular menses occurring every 30 days. They both report no history of sexually transmitted infections. Neither individual has a family history of infertility or genetic diseases. Neither of them smoke or use illicit drugs, although they have an occasional drink with dinner. Which diagnostic study is most appropriate, according to this history? AHysterosalpingography and scrotal ultrasound BHysteroscopy and semen analysis CScrotal ultrasound and semen analysis DSemen analysis

Hysteroscopy and semen analysis

A 14-year-old girl presents to the clinic with lower abdominal cramping that started six months ago. The cramping always starts a day or two before her menstrual cycle and resolves after about two days. She rates the pain as "severe"; she has gone home from school and missed sports practices due to the pain. Which of the following is the most appropriate initial therapy?

Ibuprofen non-steroidal anti-inflammatory drugs (NSAIDs) or combined oral hormonal contraceptives. NSAID

A 20-year-old woman with a known history of chronic abnormal uterine bleeding presents to the emergency department with tachycardia, hypotension, heavy vaginal bleeding, and hemoglobin of 6.2 g/dL. Pregnancy test is negative. The patient was started on fluids and given a blood transfusion. A uterine curettage was performed, but bleeding still persisted. Which of the following is the most appropriate next step?

Intravenous estrogen dx:Acute abnormal uterine bleeding cause Patient was still hemodynamically Stable In a patient with signs of hypovolemia and hemodynamic instability, fluid resuscitation and blood transfusion along with intrauterine tamponade performed to stabilize the patient. Uterine curettage is the treatment of choice in hemodynamically unstable patients with heavy uterine bleeding. If bleeding persists, the next step in treatment would be the use ofintravenous conjugated equine estrogen. Estrogen in high doses is used to reduce sudden, heavy uterine bleeding by stabilizing the endometrial lining and stopping the bleeding within one to two hours.

A 32-year-old G2P2 woman presents to the office due to left breast tenderness. She is 2 months postpatrum and had a normal vaginal delivery of a healthy baby girl. Her infant is entirely breastfed and has been gaining weight appropriately. Other than left breast tenderness and some redness that she noticed shortly before the appointment, she feels well. She has a history of mastitis with her first pregnancy. Her vital signs show a blood pressure of 119/77 mm Hg, pulse of 88 bpm, temperature of 98.6°F, and respirations of 10 breaths per minute. On physical exam, her left breast is engorged and has a 1.0 cm area of erythema and tenderness on the lateral aspect of the breast. There is no edema or dimpling of the skin. Milk is easily expressed from the left side. Which of the following is the most appropriate initial management for this patient while she continues to completely empty the involved breast?

Ibuprofen 800 mg PO as needed dx: Lactational mastitis If symptoms last longer than 24 hours, antibiotic therapy is indicated. Available antibiotic options include dicloxacillin, cephalexin, clindamycin, and trimethoprim-sulfamethoxazole. symptomatic treatment with nonsteroidal inflammatory drugs and cold compresses. In this case, ibuprofen 800 mg PO as needed is an appropriate treatment option. Additionally, complete emptying of the breast by breastfeeding, pumping, hand expression, or any combination of these techniques is recommended.

Which of the following is an immune-mediated therapy used to treat human papillomavirus?

Imiquimod

Activation of the hypothalamic-pituitary-adrenal axis through excess fetal or maternal stress has been implicated as one causative factor of preterm labor. Which of the following best describes the physiologic cascade triggered by stress in a pregnant patient?

Increased cortisol leads to increased corticotropin-releasing hormone, which activates prostaglandins that cause cervical change and rupture of membranes

Which of the following is associated with a decreased risk for endometriosis?

Increasing consumption of long-chain omega-3 fatty acids multiple childbirths, extended periods of lactation, and use of oral contraception may also decrease risk. Some protective factors include race, late menarche, and early menopause.

A 54-year-old woman presents with progressively worsening vaginal irritation and dyspareunia for three years. She denies dysuria or urinary frequency. She has no past medical history of abnormal cervical cytology, and hasn't had any vaginal bleeding in the past two years. She hasn't tried any over the counter treatments for her symptoms. There is introital narrowing on external genital exam, and pelvic exam reveals dry, smooth and shiny vaginal epithelium. What is the best next step in management?

Initiate non-hormonal vaginal moisturizer and lubricant therapy dx:Genitourinary syndrome of menopause (or atrophic vaginitis) First-line therapy for symptomatic relief is with non-hormonal vaginal moisturizers and lubricants. If therapy does not result in symptom relief, low-dose vaginal estrogen (insert, ring, cream) therapy may be used if the woman has no contraindications Estrogen therapy (A) is the most effective therapy for genitourinary syndrome of menopause, but due to potential for adverse effects of systemic absorption, including increased risk for venous thromboembolism and breast cancer, estrogen therapy is not first-line therapy.

A 52-year-old woman presents to her gynecologist with reports of vaginal pressure, dyspareunia, urinary retention, and the sensation that something is falling out of her vagina. Her urinalysis is unremarkable. Speculum examination reveals an anterior vaginal wall prolapse. Which of the following physical examination findings is associated with a risk factor for the most likely diagnosis? AChandelier sign BJoint hypermobility CScar from a previous tube thoracostomy DThin body habitus

Joint hypermobility dx cystocele is a type of pelvic organ prolapse . Risk factors for pelvic organ prolapse include vaginal birth, obesity, prior pelvic surgery, advancing age, heavy lifting or straining, genetic predisposition, and connective tissue disorders. Joint hypermobility is associated with certain connective tissue disorders such as Ehlers-Danlos syndrome, which is associated with an increased risk for developing a systole Signs and symptoms associated with a cystocele include a sensation of vaginal fullness or pressure, urinary or fecal incontinence, urinary frequency, urinary retention, increased frequency of urinary tract infections, constipation, sexual dysfunction, and dyspareunia. Diagnosis of a cystocele can be made on physical examination. Treatment of a cystocele includes activity modifications to avoid straining, Kegel exercises to strengthen the pelvic floor, pessary insertion to support the bladder in the correct place, and surgical procedures

A 52-year-old woman presents to her gynecologist with reports of vaginal pressure, dyspareunia, urinary retention, and the sensation that something is falling out of her vagina. Her urinalysis is unremarkable. Speculum examination reveals an anterior vaginal wall prolapse. Which of the following physical examination findings is associated with a risk factor for the most likely diagnosis?

Joint hypermobility dxcystocele Risk factors for pelvic organ prolapse include vaginal birth, obesity, prior pelvic surgery, advancing age, heavy lifting or straining, genetic predisposition, and connective tissue disorders. Joint hypermobility is associated with certain connective tissue disorders such as Ehlers-Danlos syndrome,(also in cervical insufficient) which is associated with an increased risk for developing a cystocele.

A 40-year-old woman presents to the clinic with concerns for a right breast lump. Physical exam reveals a 3 cm mass located in the right upper quadrant of the right breast. Mammogram reveals a 3 cm by 1.5 cm spiculated mass with notable microcalcifications. A core needle biopsy is also obtained and shows well-differentiated tumor cells that infiltrate the stroma suggestive of a stage I infiltrating ductal carcinoma. Immunohistochemical staining reveals that the mass is estrogen and progesterone receptor positive. What is the next most appropriate step in management?

LOOK INTO THIS Lumpectomy with sentinel lymph node biopsy

A 36-year-old woman presents to her gynecologist with complaints of heavy menstrual cycles and dysmenorrhea for the past 5 months. She states her periods have previously been normal. She reports no dyspareunia or dyschezia. Pelvic examination reveals the presence of an enlarged, mobile, globular, boggy uterus with no adnexal tenderness. No cervical discharge is seen on speculum exam. Her urine pregnancy test is negative. Transvaginal ultrasound is significant for an enlarged uterus. Which of the following treatments is most appropriate for the suspected diagnosis?

Levonorgestrel-releasing intrauterine device dx:Abnormal uterine bleeding . Adenomyosis is a condition where endometrial glands and stroma are present within the myometrium. Patients typically present with heavy menstrual bleeding and dysmenorrhea. Definitive diagnosis is made with histologic evaluation after hysterectomy. Definitive treatment is a total hysterectomy, although several uterus-conserving treatment options are available. A levonorgestrel-releasing intrauterine device is the recommended pharmacologic treatment, though depot gonadotropin-releasing hormone analogs or aromatase inhibitors may also be used. Uterine artery embolization, endomyometrial ablation, and radiofrequency ablation are other options available for women with adenomyosis who may not want a future pregnancy. ================================================ Danazol (A) is a derivative of 17-alpha-ethinyltestosterone that inhibits pituitary gonadotropin secretion, ovarian estrogen production, and endometriotic implant growth and increases free testosterone levels. It is most commonly used for the treatment of endometriosis and is known for its androgenic side effects. An estrogen-progestin contraceptive (B) is the first-line treatment for most causes of abnormal uterine bleeding. However, little data to support its efficacy in the treatment of adenomyosis. Furthermore, estrogen-progestin contraceptives may increase the size of leiomyomas.Mifepristone (D) is an antiprogestin medication previously used to reduce the size of uterine fibroids. It is not currently approved by the Food and Drug Administration for fibroid use and is associated with endometrial hyperplasia and elevated serum aminotransferases.

A woman reports to the office 2 days postpartum due to persistent bloody discharge. She describes the bleeding as red-brown and states it resembles her menstrual discharge. Which of the following is the correct name and appropriate duration of this discharge?

Lochia rubra, which should subside in a few days The uterus returns to the pelvis after 2 weeks and achieves a normal, nonpregnant size after 6 weeks. Postpartum women may report menses-like bleedingfollowed by other forms of vaginal discharge, termed lochia. After the placenta separates from the uterine wall, the basal portion of the decidua remains. The superficial layer is shed while the deeper layer regenerates the endometrium. This initial shedding results in red or red-brown discharge (lochia rubra) that lasts for a few days following delivery. Lochia rubra is followed by lochia serosa, which has a pinkish-brown coloring that lasts for 2 to 3 weeks final yellowish-white discharge, termed lochia alba, is composed of serous exudate, erythrocytes, leukocytes, decidua, epithelial cells, and bacteria. 1) Telogen effluvium is the loss of hair in the resting stage and is a common concern 1 to 5 monthsfollowing delivery. Normal hair patterns are generally restored after 6 to 15 months. Immediate weight loss from delivery of the fetus and placenta and from amniotic fluid loss is approximately 13 pound hospital stay following a vaginal birth is 48 hours, while patients who undergo cesarean delivery are admitted for 72 hours following the operation. When does ovulation resume after childbirth on average? Answer: After 45 days in nonlactating women and 189 days in lactating women.

A woman reports to the office 2 days postpartum due to persistent bloody discharge. She describes the bleeding as red-brown and states it resembles her menstrual discharge. Which of the following is the correct name and appropriate duration of this discharge? post partum findings When does ovulation resume after childbirth on average?

Lochia rubra, which should subside in a few days The uterus returns to the pelvis after 2 weeks and achieves a normal, nonpregnant size after 6 weeks. Postpartum women may report menses-like bleedingfollowed by other forms of vaginal discharge, termed lochia. After the placenta separates from the uterine wall, the basal portion of the decidua remains. The superficial layer is shed while the deeper layer regenerates the endometrium. This initial shedding results in red or red-brown discharge (lochia rubra) that lasts for a few days following delivery. Lochia rubra is followed by lochia serosa, which has a pinkish-brown coloring that lasts for 2 to 3 weeks final yellowish-white discharge, termed lochia alba, is composed of serous exudate, erythrocytes, leukocytes, decidua, epithelial cells, and bacteria. 1) Telogen effluvium is the loss of hair in the resting stage and is a common concern 1 to 5 monthsfollowing delivery. Normal hair patterns are generally restored after 6 to 15 months. Immediate weight loss from delivery of the fetus and placenta and from amniotic fluid loss is approximately 13 pound hospital stay following a vaginal birth is 48 hours, while patients who undergo cesarean delivery are admitted for 72 hours following the operation. When does ovulation resume after childbirth on average? Answer: After 45 days in nonlactating women and 189 days in lactating women.

A 31-year-old woman presents to the clinic complaining of groin pain. The patient states that she returned from a trip to the Caribbean three weeks ago, and had unprotected sex with a male partner during her trip. She noticed a painless genital ulcer that appeared prior to returning home, but states that it was gone three days later. A few days ago, she noticed some pain and swelling in her groin, which has become progressively worse. On physical examination, there are inflamed superficial and deep inguinal lymph nodes forming a "groove" sign bilaterally. Which of the following is the most likely diagnosis?

Lymphogranuloma venereum Primary infection occurs 3 to 12 days following exposure, and is characterized by a genital ulcer or an inflammatory reaction of the mucosa at the site of inoculation, which heals within a few days. Secondary infectionoccurs two to six weeks later and is characterized by local direct extension of the infection to regional lymph nodes, most commonly the inguinal and femoral nodes. "groove" sign . Late infection results in fibrosis and strictures in the anogenital tract, which are a result of untreated infection. First line treatment is doxycycline 100 mg orally twice daily for 21 days. Second line treatment is erythromycin 500 mg orally four times daily for 21 days. Buboes may require needle aspiration or incision and drainage.

What are some commonly used tocolytic drugs?

Magnesium sulfate, indomethacin, terbutaline, and nifedipine.

A 36-year-old woman who is 37 weeks pregnant with her first child presents to the hospital after feeling a gush of liquid from her vaginal area and seeing clear fluid while in the shower about 2 hours ago. She was unsure what it was but has been feeling more pelvic pressure and decided to come in for an evaluation. She is afebrile and has not had any contractions. A speculum examination reveals leakage from the cervical os with pooling of fluid in the vaginal vault. Which of the following is the best next step in management for this patient?

Medical induction with oxytocin dx:Prelabor rupture of membranes (PROM) The treatment of choice in cases of PROM is medical induction with oxytocin and prompt delivery, since the risk of intrauterine infection increases with the duration of ruptured membranes. If labor or vaginal delivery is contraindicated, a caesarean section should be considered next. Waiting for spontaneous labor to begin (i.e., expectant management) not only increases chances of infection, but also increases the risk of serious complications such as cord prolapse or abruption. However, for women who choose expectant management over labor induction, a time limit should be agreed upon through the process of shared decision-making between the patient and the medical team involved. ================================================= Expectant management and delivery (B) is not the best option, since infection is a serious complication that can occur during the waiting period after prelabor rupture of membranes has occurred.

A 26-year-old woman is newly diagnosed with gestational diabetes mellitus at 26 weeks gestation. Her past medical history is otherwise benign and she is on no chronic medications. Which of the following is the most appropriate first-line treatment for this patient?

Medical nutritional therapy dx:Gestational Diabetes Mellitus Screening between 24 and 28 weeks with 50 g glucose load (abnormal: glucose > 130-140 mg/dL after 1 hr) Diagnostic criteriaA 100 g glucose challenge with > 95 mg/dL fasting, > 180 mg/dL at 1 hour, > 155 mg/dL at 2 hours, or > 140 mg/dL at 3 hours> 130-140 mg/dL after 1 hour challenge can be positive depending on facility and local prevalence Rx: lifestyle changes, fetal growth monitoring, insulin Fetal risks: macrosomia, respiratory distress syndrome, neonatal hypoglycemia ↑ maternal risk of type 2 DM Which type of insulin preparation is the least immunogenic and minimizes transplacental transport of insulin antibodies? Answer: Human insulin preparations.

A 32-year-old woman presents with a complaint of increased menstrual bleeding for the past several months. She reports that her periods used to be moderate flow for 2-3 days. However, now they last for 7-10 days, and she is changing her pad or tampon every hour. Her pelvic exam is unremarkable, and no blood is seen at this time. Endometrial biopsy shows hyperplasia with atypia. A dilation and curettage reveals no evidence of endometrial carcinoma. The patient is very concerned because she has been trying to get pregnant. Which of the following is recommended for this patient?********

Megestrol acetate dx AUB tx endometrial hyperplasia!!!!!

A 32-year-old woman presents with a complaint of increased menstrual bleeding for the past several months. She reports that her periods used to be moderate flow for 2-3 days. However, now they last for 7-10 days, and she is changing her pad or tampon every hour. Her pelvic exam is unremarkable, and no blood is seen at this time. Endometrial biopsy shows hyperplasia with atypia. A dilation and curettage reveals no evidence of endometrial carcinoma. The patient is very concerned because she has been trying to get pregnant. Which of the following is recommended for this patient?

Megestrol acetate and depot medroxyprogesterone acetate are approved treatments dx endometrial hyperplasia risk factors for this condition Conditions with chronically high estrogen levels include nulliparity, late menopause, obesity, early menarche, polycystic ovarian syndrome, metabolic syndrome, chronic anovulation, estrogen producing tumors, and unopposed estrogen replacement therapy.

A 23-year-old woman presents to the office complaining of lower abdominal pain for the last 2 days. The pain is worse on the left, and she cannot seem to find a comfortable position. Her last menstrual period was 2 months ago, and her menses are usually every 28 days. She reports very light spotting on day 32 of her cycle. Her vital signs show a blood pressure of 121/78 mm Hg, pulse of 94 bpm, temperature of 98.6°F, and respirations of 10 breaths per minute. An abdominal exam reveals tenderness in the left lower quadrant. Bimanual exam reveals a 6 cm uterus with smooth contours. There is adnexal fullness on the left. The cervix is closed and without abnormalities. A urine human chorionic gonadotropin is positive, and serum value is 2,000 mlU/L. A transvaginal ultrasound reveals a left adnexal mass of 1.8 cm with a small amount of free fluid in the cul-de-sac. What is the most appropriate management option at this time?

Methotrexate dx:ectopic pregnancy vary depending on gestational age and patient stability. For stable patients, methotrexate is the initial therapy of choice, pending the adnexal mass is ≤ 3-4 cm and serum hCG values are < 5,000 mIU/L. Following administration of methotrexate, the patient is monitored with serial hCGs until the values are 0 mIU/L, thus the patient must be able to follow up. plus willing to comply and no fetal cardiac activity For unstable patients, patients with an adnexal mass > 3-4 cm, or those with serum hCG values > 5,000 mIU/L, surgical management with laparoscopic salpingostomy is the management of choice. Without treatment, an ectopic pregnancy can lead to fallopian tube rupture and severe hemorrhage. For this reason, it is considered an emergentcondition and should be managed appropriately.

Which of the following is a first-line treatment for this condition B.v?

Metronidazole 250 mg orally three times daily for 7 days

Which of the following factors in the patients history increases her risk for cervical cancer?

Multiple sex partners HPV 16.

A 35-year-old woman, who is currently breastfeeding, presents with left breast tenderness, redness, and warmth. On exam, you note a fluctuant, tender, and palpable left breast mass without ischemia or necrosis. Breast ultrasound demonstrates a fluid collection within the left breast. Which of the following is the most appropriate clinical intervention?

Needle aspiration, antibiotics, and continuing breastfeeding dx:breast abscess -----Surgical drainage may be indicated if needle aspiration and antibiotic therapy are not successful or if the overlying skin is compromised.-------

A 24-year-old woman presents with sudden onset of lower right sided abdominal pain, nausea, and vomiting. She is not currently sexually active and pregnancy test is negative. She is found to have a right adnexal mass on exam. A pelvic ultrasound reveals an heterogenous enlarged ovary with decreased doppler flow. Which of the following is the most common risk factor for the suspected diagnosis?

Ovarian mass dx:Ovarian torsion the ovary typically rotates around which ligament or ligaments when torsion occurs? Answer: Both the infundibulopelvic ligament (suspensory ligament of the ovary) and the utero-ovarian ligament.

A 23-year-old woman presents with several erythematous, painful ulcers on the vaginal introitus which have been present for seven days and began as small red papules, which then evolved to pustules and then ulcerations. There is marked lymphadenopathy of bilateral inguinal nodes. Polymerase chain reaction for herpes simplex virus is negative. Which of the following additional diagnostic tests is necessary before diagnosing chancroid in this patient?

Negative venereal disease research laboratory test dx:Chancroid Haemophilus ducreyi, a gram-negative rod Chancroid begins as one or several erythematous papules on areas of the genitals most susceptible to friction, such as the glans penis or the vaginal introitus. The papules become pustules which ulcerate and form painful open sores, usually of one to two centimeters in diameter. The ulcers have an erythematous base covered by a gray or yellow purulent exudate marked lymphadenopathy in the inguinal chain What is a bubo? Answer: Fluctuant lymphadenitis associated with genital infections such as chancroid. chancroid cases are diagnosed based on the presence of all four of the following criteria: 1). one or more painful genital ulcers, a negative polymerase chain reaction (or culture) for herpes simplex virus, 2) no evidence of Treponema pallidum on darkfield microscopy of the ulcer exudate or a negative venereal disease research laboratory test (or other serologic test for syphilis), 4) and clinical signs and symptoms consistent with chancroid. Treatment of chancroid is with ceftriaxone 250 mg injected intramuscularly or with one gram of oral azithromycin.

A 27-year-old G2P1001 woman presents to labor and delivery at 33 weeks and 2 days of gestation with regular uterine contractions for 2 hours. She reports no vaginal bleeding or clear leakage of fluid. Pelvic examination reveals cervical dilation to 3 cm. Which of the following is the best recommendation for tocolytic therapy? AIndomethacin BNifedipine CTerbutaline DTocolytic therapy is not indicated

Nifedipine dx: Preterm labor Antenatal corticosteroids and tocolytic therapy are not used after 34 weeks gestation Women at ≥ 34 weeks gestation with preterm labor are admitted for delivery. Which drug is considered the first-line corticosteroid during preterm labor? Answer: Betamethasone. The recommended treatment for women with suspected preterm labor based on the above criteria involves tocolytic therapy, antenatal corticosteroids, antibiotic group B streptococcal prophylaxis[screened between 36-37 weeks gestation.] (if indicated), and magnesium sulfate.

An asymptomatic G2P2 58-year-old woman presents to the clinic for routine gynecologic examination. Pelvic exam reveals a posterior vaginal bulge and a palpable break in the rectovaginal fascia. Which of the following interventions is the best next step?

Observe with yearly examination dxRectocele is the anterior protrusion of the rectum into the posterior vaginal wall. Observation with yearly examinations is appropriate management for asymptomatic women or for women with mild symptoms and a mild prolapse because the majority of these women may never experience prolapse progression Can pelvic organ prolapse occur in nulliparous women? Answer: Yes.

A 41-year-old G1P0 woman presents for a routine prenatal visit. She is 35 weeks pregnant. She reports a normal pregnancy course to date and feels well today. She has experienced a 25 lb weight gain over the last 8 months and reports recent lower extremity swelling. The swelling usually resolves when she elevates her feet at night but not always. Her bowel movements have been normal, but she reports frequent urination and lower abdominal pain. The pain is occasional, sharp, and fleeting. It usually occurs after sudden movements. She also reports visual flashes of light, which are infrequent but occur daily. She reports neither eye pain nor blurry vision. Upon arrival, her vital signs show a blood pressure of 134/92 mm Hg, pulse of 80 bpm, temperature of 98.6°F, and respirations of 12 breaths per minute. Pupils are equal, round, and reactive to light and accommodation bilaterally. Cardiovascular exam reveals a regular rate and rhythm without murmurs, rubs, or gallops. The lungs are clear to auscultation bilaterally. Her abdomen measures 35 cm from the pubic symphysis to the uterine fundus. There is no tenderness on palpation. She has 2 pitting edema bilaterally at her ankles and distal lower legs. The fetal heart rate is 152 bpm. A routine urinalysis is pending. Which aspect of this patient's history and physical exam is most concerning? AElevated diastolic blood pressure BLower extremity edema COccasional flashes of light in her vision DSharp lower abdominal pain

Occasional flashes of light in her vision dx: preeclampsia What disorder is associated with preeclampsia that presents prior to 20 weeks of gestation? Answer: A molar pregnancy. Preeclampsia Pregnancy > 20 weeks gestation or postpartum Visual disturbances, severe headaches, or asymptomatic Evaluation will show new-onset hypertension (≥ 140/90 mm Hg) with either proteinuria (≥ 300 mg/24 hr or urine protein: creatinine ratio ≥ 0.3) OR significant end-organ dysfunction Treatment: delivery at 37 weeks (without severe features) and 34 weeks (with severe features) AND prevention of seizures with magnesium sulfate and prevention of permanent maternal organ damage New-onset hypertension < 20 weeks gestation: suspect molar pregnancy

A 38-year-old G3P2 at 16 weeks' gestation presents to the clinic for a regular prenatal visit. She describes symptoms of morning sickness that are "worse than with my other two pregnancies." Vitals are normal. On exam, her fundal height measured 18 cm. Labs show an elevation in human chorionic gonadotropin hormone and maternal serum alpha-fetoprotein. Which of the following is the next best step?

Order abdominal ultrasound dx:Multiple gestations has become more common due to older maternal age ypical presentation may include: increased uterine size and growth rate, excessive maternal weight gain, and morning sickness. Laboratory studies may reveal increased levels of human chorionic gonadotropin hormone and maternal serum alpha-fetoprotein for gestational age.

A 32-year-old woman who is 35 weeks pregnant presents for a routine prenatal examination. Upon palpation of the uterus, the firm resistance of the fetal head is not able to be felt above the symphysis pubis. Upon further palpation, the fetal head seems to be located in the left upper quadrant. Which of the following is the next best step?

Order an abdominal ultrasound External version is typically attempted at 37 weeks gestation which allows for ample amniotic fluid while restricting the time period for recurrence. What is the most common risk factor for transverse lie at birth? Answer: Prematurity.

A 22-year-old woman in the Caribbean presents to the public health clinic with buboes in her right inguinal area. She had unprotected intercourse with a new sexual partner 6 weeks ago. Which of the following clinical manifestations is most commonly seen in primary infection with this condition? ********

Painless genital ulcer dx : Lymphogranuloma venereum is a genital ulcer disease caused by specific serotypes of Chlamydia trachomatis Lymphogranuloma Venereum Primarily seen in men who have sex with men History of recent travel to tropical and subtropical areas of the world Small, shallow painless genital ulcer PE will show tender inguinal or femoral lymphadenopathy Most commonly caused by Chlamydia trachomatis Treatment is doxycycline Primary infection presents with a painless and small genital ulcer at the site of inoculation. The painless ulcer resolves spontaneously within a few days and thus often goes unnoticed. Secondary infection occurs 2 to 6 weeks following the primary infection. The secondary stage is due to inflammation from direct extension to the superficial and deep inguinal or femoral lymph nodes. The secondary stage may manifest as an inguinal syndrome or as anorectal symptoms. The unilateral painful inguinal lymph nodes associated with lymphogranuloma venereum are often called buboes What is the recommended treatment for patients who cannot take doxycycline? Answer: Azithromycin once weekly for 3 weeks.

A 22-year-old woman in the Caribbean presents to the public health clinic with buboes in her right inguinal area. She had unprotected intercourse with a new sexual partner 6 weeks ago. Which of the following clinical manifestations is most commonly seen in primary infection with this condition?

Painless genital ulcer dx: Lymphogranuloma venereum Primary infection presents with a painless and small genital ulcer at the site of inoculation. The painless ulcer resolves spontaneously within a few days and thus often goes unnoticed. The secondary stage is due to inflammation from direct extension to the superficial and deep inguinal or femoral lymph nodes. The secondary stage may manifest as an inguinal syndrome or as anorectal symptoms. The unilateral painful inguinal lymph nodes associated with lymphogranuloma venereum are often called buboes. What is the recommended treatment for patients who cannot take 21-day course doxycycline? Answer: Azithromycin once weekly for 3 weeks. The buboes are sometimes drained for symptomatic relief. It is important to have all sexual partners of the patient evaluated and treated and to have the patient tested for hepatitis B, hepatitis C, and HIV infection.

27-year-old nulliparous woman presents to her gynecologist for a routine examination. She voices no concerns at this time. She is sexually active with three male partners and does not use any form of contraception. Her Pap smear report returns atypical squamous cells, cannot exclude high-grade squamous intraepithelial lesion. During colposcopy, a white, sharply demarcated lesion on the cervix is visualized after acetic acid is applied. Colposcopy is adequate, and cervical biopsy results reveal cervical intraepithelial neoplasia grade 2. The patient states she would like to have children in the future and the risks of treatment outweigh the risk of cancer. Which of the following is the best plan for management?

Pap smear and colposcopy in 6 months who consider the risks of treatment to outweigh the risks of cancer may be observed with a Pap smear and colposcopy in 6 months and 12 months. If the Pap smear and colposcopy are negative at both of these intervals, a Pap smear and human papillomavirus (HPV) co-testing 1 year later is recommended. If co-testing is negative, co-testing should be repeated 3 years later. If co-testing is abnormal, a colposcopy should be done. If colposcopy results reveal worsening or high-grade lesions, a repeat biopsy should be done. If CIN 2 or 3 persists for more than 2 years, treatment is recommended. If colposcopy is inadequate, if the patient has CIN 3, or if the patient has CIN 2 and opts for treatment, excision or ablation of the cervical transformation zone is indicated. Excisional treatments include loop electrosurgical excision procedure and cold knife conization.Ablative treatments include cryotherapy and laser therapy.

A 45-year-old woman presents to the office complaining of a new vulvar lesion. She reports the lesion first appeared 2 weeks ago and has become slightly larger. It is not tender and does not bleed. Her past medical history is significant for anxiety, depression, and HIV. She has no drug allergies. Her most recent CD4+ count was 750 cells/microL. She has a history of risky sexual behavior and reports four new sexual partners in the last 2 months. She prefers not to use condoms. On physical exam, there is a 1.1 cm shallow, painless, erythematous ulceration on the mucosal surface of her left labia. Serology testing, including a CD4+ count, is ordered, and results are pending. In the meantime, what is the most appropriate pharmacotherapeutic option for this patient?

Penicillin 1 dose of 2.4 million units dx: syphillis Which sexually transmitted infection is associated with the congenital abnormality known as Hutchinson teeth? Answer: Syphilis.

A 28-year-old woman presents with a persistent painless labial lesion for one week. She denies dysuria or any change in her vaginal discharge. On exam, she has one erythematous ulcerated sore on the labia majora with a punched-out appearance. The rest of physical exam is unremarkable. Which of the following is the most appropriate therapy? What is the recommended treatment for early stage of this disease in a penicillin-allergic, non-pregnant patient?

Penicillin G benzathine IM dx:Syphilis What is the recommended treatment for early syphilis in a penicillin-allergic, non-pregnant patient? Answer: Doxycycline.

A 67-year-old G6P5 woman presents to the clinic complaining of pelvic pressure that has progressively worsened for months. She also complains she cannot empty her bladder completely. She is currently sexually active. Physical examination reveals collapse of the anterior vaginal wall. Which of the following is the recommended initial treatment?

Pessary

A 24-year-old G1P0000 woman presents to the clinic at 36 weeks gestational age for a routine prenatal visit. Her blood pressure is noted to be 142/102 mm Hg and is 146/104 mm Hg on repeat measurement 4 hours later. She has no prior history of hypertension. Which of the following additional findings would support a diagnosis of severe preeclampsia?

Photopsias dx= preeclampsia criteria : proteinuria or end-organ features that develop after 20 weeks blood pressure ≥ 140/90 mm Hg on two occasions≥ 4 hours apart Proteinuria ≥ 300 mg in a 24-hour urine collection what made it severe severe headache or headache that does not resolve with acetaminophen, visual disturbances (e.g., photopsia, scotomata), elevated creatinine, elevated liver function tests, thrombocytopenia, pulmonary edema, or abnormal fetal findings (e.g., growth restriction, oligohydramnios, evidence of fetal distress). The risk factors for preeclampsia include a personal or family history of preeclampsia, nulliparity, chronic hypertension, pregestational diabetes mellitus, and multiple gestation. thrombocytopenia <100,000 tx:Antenatal corticosteroids, such as betamethasone, are administered to women with preeclampsia who are < 34 weeks pregnant. Women with preeclampsia should be administered intrapartum and postpartum seizure prophylaxis with magnesium sulfate. Which medication can be offered to women with risk factors for preeclampsia to prevent preeclampsia? Aspirin.

A 24-year-old woman presents to the emergency department after delivering a baby at a birthing center about 1 hourago. She is pale and fatigued, with a blood pressure of 92/54 mm Hg and a heart rate of 124 beats per minute. She is wearing a pad that she states has been changed multiple times within the past hour and has been filled with bright red blood and clots. Which of the following placental complications has the greatest risk for postpartum hemorrhage due to the depth of invasion into the uterine myometrium?

Placenta percreta Postpartum hemorrhage

A 23-year-old woman presents to her gynecologist for routine prenatal care. She is at 35 weeks gestation with her first pregnancy. She reports ongoing epigastric pain, which she associates with reflux, and in the last week, she has had a persistent headache. Her vital signs are concerning for a blood pressure of 150/100 mm Hg. Her urine dipstick is negative for protein. She is advised to return the next day for repeat evaluation, including vital signs and additional laboratory tests. On her return visit, her blood pressure is unchanged. Which of the following laboratory findings confirms the diagnosis?

Platelets < 100,000/µL dx: preeclampsia severe presentations may present without proteinuria but with signs of end-organ dysfunction. Preeclampsia is unique to pregnancy and increases the potential for the development of placental and maternal vascular dysfunction Preeclampsia falls into two categories: with or without severe features. To meet the criteria for diagnosis, women in both categories must have blood pressure > 140/90 mm Hg on two occasions at least 4 hours apart, or 160/110 mm Hg on one occasion. Women with preeclampsia without severe features must also have proteinuria > 300 mg over 24 hours or a urine protein to creatinine ratio > 0.3 mg/dL. Preeclampsia with severe features is diagnosed by the same blood pressure criteria and at least one additional symptom or laboratory finding indicative of end-organ dysfunction, namely pulmonary edema, new-onset cerebral or visual disturbances, severe and persistent right upper quadrant orepigastric pain, thrombocytopenia (platelets < 100,000/µL), progressive kidney insufficiency (creatinine concentration > 1.1 mg/dL), or serum transaminase two or more times the upper limit of normal. If these conditions exist, a negative proteinuria finding does not change the diagnosis of preeclampsia with severe features Development of grand mal seizures in a woman with preeclampsia is classified as eclampsia, and HELLP syndrome(hemolysis, elevated liver enzymes, low platelets) is recognized as a subtype of preeclampsia in which these features may present with or without hypertension or proteinuria. The only definitive treatment to prevent complications to mother and baby is delivery.

A 24-year-old G2P1 woman at 24 weeks gestation presents to her obstetrician for a routine visit. Her blood glucose for the 50 gram 1-hour glucose challenge test is 185 mg/dL. A 100 gram 3-hour glucose tolerance test is ordered, and her blood glucose is 110 mg/dL fasting, 205 mg/dL at 1 hour, 180 mg/dL at 2 hours, and 165 mg/dL at 3 hours. Her glucose challenge test was negative during her last pregnancy. Which of the following complications is associated with the patient's current condition?

Polyhydramnios Complications associated with gestational diabetes include preeclampsia, gestational hypertension, polyhydramnios, large for gestational age infant, maternal and infant birth trauma, perinatal mortality, and neonatal complications (e.g., shoulder dystocia, hypertrophic cardiomyopathy, hypoglycemia, hyperbilirubinemia, hypocalcemia, polycythemia).

A 24-year-old woman at 37 weeks gestation presents to the office for a routine obstetrics exam. An ultrasound reveals an amniotic fluid index of 4 cm and no signs of fetal abnormalities. Which of the following is a risk factor for this condition? AFetal esophageal atresia BFetal hyperglycemia CPreeclampsia DTrisomy 21

Preeclampsia look at image !!!! dx:Oligohydramnios which can lead to underdevelopment of fetal lung tissue as well as fetal death. #barrier against infection Oligohydramnios can be evaluated by ultrasonography, which will show an amniotic fluid index < 5 cm or a single deepest pocket that is < 2 cm in depth. Women with diagnosed oligohydramnios should be put on bed rest with adequate hydration to promote the production of amniotic fluid. A thorough evaluation of the fetal anatomy should be done, as well as genetic testing as indicated. most common cause of oligohydramnios is the rupture of membranes When the fetus is near term, it swallows amniotic fluid and produces urine. Therefore, a low amniotic fluid level can be caused by the absence of fetal urine production or fetal urinary tract blockage. Any anomaly that affects the fetal urinary tract, such as renal agenesis, polycystic kidneys, or obstructive lesions, can lead to this condition. It can also be caused by placental insufficiency due to conditions such as preeclampsia, chronic hypertension, and other maternal vascular disease or when the pregnancy is extended too long

Which of the following maternal characteristics is a risk factor for preterm labor?

Pregnant patients with type 1 diabetes, hypertension, thyroid disease, asthma, kidney insufficiency, nonphysiologic anemia, major depressive disorder, and certain autoimmune disorders may be predisposed to preterm labor. Certain infections during pregnancy can also contribute to preterm labor, such as gonorrhea, chlamydia, bacterial vaginosis, trichomoniasis, syphilis, urinary tract infection, pyelonephritis, endometritis, and some severe viral infections. Ingestion, injection, or inhalation of alcohol, cocaine, heroin, and tobacco are also predisposing factors for preterm labo

Which of the following factors is more indicative of premenstrual dysphoric disorder than premenstrual syndrome?

Presence of symptoms for most of the preceding year

A 28-year-old G2P1 woman at 23 weeks gestation presents to the emergency room for painless vaginal bleeding. Prior to performing a digital vaginal examination, an ultrasound of the uterus is performed, which reveals placental tissue lying over the internal cervical os. Based on the ultrasound findings, what would you expect to learn from the patient's history that would put her at increased risk of the likely diagnosis?

Previous cesarean section dx: placenta previa What other fetal and placental complications are associated with placenta previa? Answer: Malpresentation, intrauterine growth restriction, congenital anomalies, vasa previa, and velamentous umbilical cord.

Which of the following is the strongest risk factor for recurrence of ectopic pregnancy? Women with a history of intrauterine exposure to which drug have an increased risk of ectopic pregnancy?

Previous history of ectopic pregnancy Women with a history of intrauterine exposure to which drug have an increased risk of ectopic pregnancy? Answer: Diethylstilbestrol.

A 33-year-old G2P1 woman presents to the emergency department reporting severe abdominal pain and dark red vaginal bleeding. She is currently 37 weeks pregnant and appears to be in labor. On exam, the baby is bradycardic, and a bedside ultrasound shows separation of the placenta edge. What is the greatest risk factor for this patient's condition?

Previous placental abruption

A 30-year-old nulliparous woman presents to her gynecologist with prolonged and heavy menstrual bleeding for the past 6 weeks. Upon further questioning, she states she has also had abdominal discomfort. She is not currently taking any medications. She is sexually active with three male partners and does not use contraception. Pelvic examination is significant for a mildly enlarged uterus (7 cm) without cervical motion tenderness or adnexal masses. Transvaginal ultrasound confirms the suspected diagnosis. Which of the following treatments is most appropriate for the patient at this time?

Progestin-releasing intrauterine device dx:Leiomyomas, also known as uterine fibroids, --------------------------------------------------- Combined estrogen-progestin contraceptives, a progestin-releasing intrauterine device, and tranexamic acid are considered first-line pharmacologic agents. Intrauterine devices can be used in patients whose uterus size is between 6 and 10 cm and without a large submucosal component. Second-line treatments include gonadotropin-releasing hormone agonists( Leuprolide acetate and goserelin acetate) and antagonists and uterine artery embolization, both of which decrease fibroid size and reduce bleeding. For patients desiring fertility, myomectomy is the first-line treatment. Depending on the location, a hysteroscopic, laparoscopic, or open abdominal myomectomy may be performed. Patients who do not desire fertility may be treated with a hysterectomy.

Which of the following factors is more indicative of secondary dysmenorrhea than primary dysmenorrhea?

Progressive worsening of pain severity

A 15-year-old girl presents with a recurrent, crampy lower abdominal pain, nausea, headache, and malaise. Her symptoms usually start 1 to 2 days before the onset of menstrual bleeding and then gradually diminishes over 12 to 72 hours. On exam, you note tenderness to palpation of the lower abdomen. Laboratory studies and pelvic ultrasound show no abnormal findings. Which of the following is the most likely cause of this patient's symptoms?

Prostaglandin production dx: Dysmenorrhea

Which ligaments support the uterus and attach the cervix to the posterior surface of the pubic symphysis?

Pubocervical ligaments.

A 24-year-old woman at 14 weeks gestation reports to the clinic for her second prenatal visit. She reports intense nausea with occasional vomiting that has not resolved with dietary changes, including the addition of ginger and peppermint teas. Which of the following is the best next step in treatment?

Pyridoxine Prenatal care n/v management in the image Question: What tool can be used to quantify the severity of nausea and vomiting in pregnancy? Answer: Pregnancy Unique-Quantification of Emesis (PUQE) score. Doxylamine can be added when pyridoxine alone is ineffective. Medical therapy should progress in a stepwise fashion thereafter with use of histamine H1 antagonists (e.g., dimenhydrinate, diphenhydramine, meclizine), dopamine antagonists (e.g., metoclopramide, promethazine, prochlorperazine), and serotonin antagonists (e.g., ondansetron, granisetron). Acid-reducing therapies, such as antacids, histamine H2 receptor antagonists, and proton pump inhibitors, can be added as adjunct agents. Refractive nausea and vomiting should be further evaluated with laboratory testing, including Helicobacter pylori testing, and can be treated using glucocorticoids and the dopamine antagonists chlorpromazine and droperidol. These medications have less desirable side effect profiles and may carry increased maternal and fetal risks compared to the medications described above. F

1)At what age should routine screening for osteoporosis begin for women according to the United States Preventive Services Task Force? Answer: 65 years of age. 2)Which intravenous medications are used to acutely lower hypertension in pregnant patients? Answer: Hydralazine and labetalol. 3)What is the term for hemolytic disease of the newborn when the infant is still in utero? Answer: Erythroblastosis fetalis. 4) What is primary ovarian insufficiency? Answer: Hypergonadotropic hypogonadism before the age of 40 years. 5)True or false: brachial plexus injuries from shoulder dystocia are usually permanent. Answer: False, most result in no long-term deficits. 6)What disorder is associated with preeclampsia that presents prior to 20 weeks of gestation? Answer: A molar pregnancy. 7) Bacterial vaginosis tx Metronidazole 250 mg orally three times daily for 7 days 8)What are some differential diagnoses of bacterial vaginosis? Answer: Vulvovaginal candidiasis and trichomoniasis. 9)True or false: cystocele, rectocele, enterocele, and urethrocele are all vaginal hernias. Answer: False. A urethrocele is not a vaginal hernia. 10)What is a theca lutein cyst? Answer: An ovarian cyst that results from overstimulation by beta-human chorionic gonadotropin, such as occurs during molar pregnancy, multiple gestation, or clomiphene therapy. 11)Which muscular fibers converge at the perineal body? Answer: Deep and superficial transverse perineal muscles, bulbocavernosus and pubococcygeus muscles, and the external anal sphincter. 12)What is the recommended treatment for patients who cannot take doxycycline? Answer: Azithromycin once weekly for 3 weeks. 13)True or false: an individual who has had one preterm birth is at increased risk for preterm labor in subsequent pregnancies. Answer: True. 14)What condition is FDA approved for treatment with anastrozole? Answer: Breast cancer. 15)What outdated technique for measuring fetal anemia has largely been replaced by Doppler velocimetry? Answer: Amniocentesis to determine amniotic fluid bilirubin level. 16)What is placenta accreta? Answer: Placental attachment to the myometrium rather than the decidua, which causes the placenta not to spontaneously separate at delivery. 17)What is the genus and species name of group B Streptococcus? Answer: Streptococcus agalactiae. 18)What is the most common age group to develop a tubo-ovarian abscess? Answer: 15-25 years of age. 19)What are the three components of the levator ani muscle? Answer: The puborectalis, pubococcygeus, and iliococcygeus. These form the pelvic floor. 20)What is the mechanism of action of tranexamic acid? Answer: It prevents the conversion of plasminogen to plasmin, which results in decreased fibrinolysis. PALM-COEIN acronym to further classify and identify abnormal uterine bleeding etiologies. This acronym stands for polyp, adenomyosis, leiomyoma, malignancy and hyperplasia, coagulopathy, ovulatory dysfunction, endometrial dysfunction, iatrogenic, and not otherwise classified. Pregnancy should be excluded 21)Which risk factor is associated with recurrent breast abscesses? Answer: Smoking. 22)What is the mechanism of action of leuprolide? Answer: It is a gonadotropin-releasing hormone analogue. It acts to cause suppression of FSH and LH. 23)True or false: topical estrogen therapy helps prevent uterine prolapse. Answer: False. 24)

Q&A

1)What antibiotic medication can be used to treat breast abscesses in the absence of risk factors for methicillin-resistant Staphylococcus aureus (MRSA) in patients who are allergic to beta-lactams and cephalosporins? Answer: Clindamycin. 2)What is the appearance of a molar pregnancy on ultrasound? Answer: A "cluster of grapes" or "snowstorm" due to increased trophoblast proliferation and enlarged chorionic villi that grow in clusters. 3)What is ptyalism gravidarum? Answer: Excessive salivation during pregnancy. 4)Which bacterium is the leading cause of endometritis-associated infertility in endemic countries? Answer: Mycobacterium tuberculosis. 5)Which sexually transmitted infection is associated with the congenital abnormality known as Hutchinson teeth? Answer: Syphilis. 6)What type of breast cancer is associated with breast abscess? Answer: Inflammatory breast cancer. 7)What is an alternative antibiotic medication that can be used to treat syphilis in the setting of penicillin hypersensitivity? Answer: Tetracycline or doxycycline. 8)True or false: patients should be instructed to avoid douching, tampon use, vaginal medications, and sexual intercourse for 24 hours prior to a colposcopy. Answer: True. 9)What is the most commonly encountered form of pelvic organ prolapse? Answer: Cystocele. 10)What are the common side effects of antimuscarinic medications? Answer: Dry mouth, constipation, tachycardia, palpitations, and exacerbation of dementia. 11)True or false: the survival rate for women with low-risk gestational trophoblastic neoplasia after treatment is 100%. Answer: True. 12)True or false: the absence of an ovarian cyst on ultrasound rules out ovarian cyst rupture. Answer: False, ovarian cysts may collapse following cyst rupture. However, the absence of an ovarian cyst makes the diagnosis less likely. 13)What is interval breast cancer? Answer: Breast cancer that presents during the period between normal mammogram screenings. 14)What is the nickname for syphilis? Answer: The great imposter. 15)What organism is most responsible for toxic shock syndrome? Answer: Staphylococcus aureus. 16)Question: What birth weight is fetal macrosomia defined by? Answer: 4,000-4,500 g, regardless of gestational age. 17)What is the name of the criteria used to diagnose polycystic ovary syndrome? Answer: Rotterdam criteria. 18) What tool can be used to quantify the severity of nausea and vomiting in pregnancy? Answer: Pregnancy Unique-Quantification of Emesis (PUQE) score. 19)What is the first sign of hypermagnesemia in patients being treated with magnesium sulfate to prevent seizures? Answer: Loss of the patellar reflex. 20)True or false: cigarette smoking increases the risk of preeclampsia. Answer: False. Cigarette smoking is associated with a lower risk of preeclampsia. 21)True or false: oligohydramnios is more common than polyhydramnios. Answer: True. Fetal esophageal atresia (A), fetal hyperglycemia (B), and trisomy 21 (D) are risk factors associated with polyhydramnios

Q&A

1)True or false: all inflammatory breast cancer tumors should undergo testing for hormone receptors and human epidermal growth factor receptor 2. Answer: True. 2)What are the five components of the biophysical profile? Answer: Fetal breathing, fetal movement, fetal tone, amniotic fluid volume, and nonstress test to measure fetal heart rate. 3)What is the definition of a shortened cervix? Answer: Less than 25 mm before 24 weeks gestation. 4)Which human papillomavirus types most frequently cause genital warts? Answer: 6 and 11. 5)Which class of medications do leuprolide acetate and goserelin acetate belong to? Answer: Gonadotropin-releasing hormone agonists. 6)True or false: the gestational sac is able to be visualized before the yolk sac on transvaginal ultrasound. Answer: True. 7)What are possible complications of a breast abscess? Answer: Recurrent infection, poor cosmetic outcome, mammary duct fistula, milk fistula, and antibioma. 8)Why should indomethacin not be used for more than 48 hours in women who are in preterm labor? Answer: Use for more than 48 hours can cause premature closure of the ductus arteriosus. 9)Which human chorionic gonadotropin level is used as the discriminatory zone for when a gestational sac should be visualized on transvaginal ultrasound? Answer: 2,000 mIU/mL. 10)What is the black box warning for metformin? Answer: Lactic acidosis. 11)True or false: the volume of milk production by mothers of twins is consistently twice that of mothers of singletons. Answer: True. 12)What are the four reagents used for the Gram stain? Answer: Crystal violet, iodine, ethanol, safranin. 13)Which bacteria most frequently causes toxic shock syndrome? Answer: Staphylococcus aureus. 14)Which autosomal dominant condition is characterized by short stature, pulmonary stenosis, hypertelorism, downslanting palpebral fissures with highly arched eyebrows, strikingly blue irises, and a webbed neck? Answer: Noonan syndrome.

Q/A

1)Which intravenous medications are used to acutely lower hypertension in pregnant patients? Answer: Hydralazine and labetalol. 2)At what age should routine screening for osteoporosis begin for women according to the United States Preventive Services Task Force? Answer: 65 years of age. 3)Which medication can be offered to women with risk factors for preeclampsia to prevent preeclampsia? Aspirin. 4)What is primary ovarian insufficiency? Answer: Hypergonadotropic hypogonadism before the age of 40 years. 5)What is the medical term used to describe a lower volume of amniotic fluid than expected for gestational age? Answer: Oligohydramnios. 6)In which quadrant of the breast do breast cancers occur most often? Answer: Upper outer quadrant. 7)True or false: brachial plexus injuries from shoulder dystocia are usually permanent. Answer: False, most result in no long-term deficits. 8)What disorder is associated with preeclampsia that presents prior to 20 weeks of gestation? Answer: A molar pregnancy. 9)What are some differential diagnoses of bacterial vaginosis? Answer: Vulvovaginal candidiasis and trichomoniasis. 10): True or false: cystocele, rectocele, enterocele, and urethrocele are all vaginal hernias. Answer: False. A urethrocele is not a vaginal hernia. 11): What is the most common site of early hematogenous metastases of a choriocarcinoma? Answer: Lungs. 12)What is a theca lutein cyst? Answer: An ovarian cyst that results from overstimulation by beta-human chorionic gonadotropin, such as occurs during molar pregnancy, multiple gestation, or clomiphene therapy. 13)Which diagnostic tests are recommended immediately prior to inserting an intrauterine device? Answer: Pregnancy testing and gonorrhea and chlamydia testing. 14)Which muscular fibers converge at the perineal body? Answer: Deep and superficial transverse perineal muscles, bulbocavernosus and pubococcygeus muscles, and the external anal sphincter. 15)What is the recommended treatment for patients who cannot take doxycycline? Answer: Azithromycin once weekly for 3 weeks. 16)What is the most common reason a vaginal culture is performed when vulvovaginal candidiasis is suspected? Answer: If the patient's symptoms are resistant to azole therapy and Candida glabrata is suspected. 17)True or false: an individual who has had one preterm birth is at increased risk for preterm labor in subsequent pregnancies. Answer: True. 18)What condition is FDA approved for treatment with anastrozole? Answer: Breast cancer.

Q/A 3

A 55-year-old woman with a history of a genital human papillomavirus-16 infection presents to the clinic with vaginal pruritus and irritation. Physical exam reveals right vulvar skin lesions. Which of the following is the most likely description of the vulvar lesions for the suspected diagnosis?

Red and white ulcerative lesions

A 55-year-old woman presents to the office due to vaginal fullness and pressure that has gotten worse over the past year. She feels a bulge at the opening of her vagina especially after being on her feet for several hours. She is also having more difficulty urinating. She does not have any problems with defecation and does not report vaginal bleeding or pain. She is widowed and has six adult children who were delivered vaginally. Which of the following is the most appropriate next step in treatment?

Refer her to a specialist for a pessary fitting dx:Uterine prolapse "wording varies

A 24-year-old woman who gave birth to a premature baby girl at 30 weeks gestation presents to your office for a follow-up visit after being diagnosed with mastitis three days ago. You had prescribed her amoxicillin, but after taking it as directed, she has not had any improvement of symptoms. The patient now has a fever of 103℉ with increased pain to the left breast. Physical exam shows a severely tender fluctuant mass in the breast that is red and warm to touch. What is the most appropriate next step for treatment of this patient?

Refer patient to a surgeon for incision and drainage dx: breast abscess What disease should be suspected in a non-lactating patient who develops a breast abscess? Answer: Diabetes mellitus.

A 35-year-old woman presents to her gynecologist for a follow-up visit to discuss her human papillomavirus (HPV) screening results. Her Pap test report came back positive for low-grade squamous intraepithelial lesion, and her reflex HPV test was negative. She has never had an abnormal HPV screen in the past. Which of the following is the best next step in management?

Repeat cotesting in 12 months (look at management )

A 26-year-old woman presents to the ED with sudden-onset pelvic pain, nausea, and vomiting for the past 2 hours. She reports no vaginal discharge or vaginal bleeding. Her medical history is significant for polycystic ovary syndrome. Her serum human chorionic gonadotropin is negative. Which of the following additional physical examination findings is most consistent with the suspected diagnosis?

Right-sided adnexal mass dx: ovarian torsion

A 24-year-old primigravid woman presents to the clinic with excessive nausea and vomiting for the past several weeks. Her last menstrual period was 2 months ago, and she states she had a positive home pregnancy test this week. Pelvic ultrasound reveals a 2 cm uterine mass with anechoic areas, increased blood flow, and myometrial invasion. No fetus is noted. Serum human chorionic gonadotropin level is 90,000 mIU/mL. Positron emission tomography reveals disease involving the uterus only. Which of the following is the appropriate clinical intervention for this patient?

Single-agent chemotherapy dx:Gestational trophoblastic disease includes benign conditions, such as hydatidiform mole, and neoplastic disease, such as choriocarcinoma, trophoblastic tumors, and invasive mole. Risk factors for the development of gestational trophoblastic disease include prior molar pregnancy and maternal age over 40 years.

A 30-year-old woman presents to the clinic with concerns about preterm delivery. She suffered premature rupture of membranes with her first pregnancy. Which of the following would be the best choice for anticipatory guidance in this patient?

Smoking cessation

A 20-year-old woman presents to the office due to abdominal bloating and swelling in her legs and feet that coincides with her menstrual cycle. She starts having some abdominal and pelvic cramping around 1 week before her period, and it gets worse until her period starts. She states these symptoms have been getting worse over the past 6 months, and she has to wear bigger clothes and bigger shoes every month when she is on her period. Which of the following is the best regimen for the patient's symptoms?

Spironolactone dx:Premenstrual syndrome (PMS)

A 60-year-old woman presents to her gynecologist with postmenopausal bleeding. She is currently taking tamoxifen following recent breast cancer treatment and is on estrogen replacement therapy for symptoms of menopause. An endometrial biopsy shows Stage 1 adenocarcinoma. Which of the following is the most appropriate intervention for this patient?

Stage 1 cancers are treated with hysterectomy and bilateral salpingo-oophorectomy with or without radiation. Stage 2 cancers are treated with hysterectomy and bilateral salpingo-oophorectomy, lymph node excision and radiation. The depth of the tumor determines prognosis.

A 62-year-old woman presents to her gynecologist with reports of vaginal heaviness and sensations of incomplete bladder emptying. A pelvic examination reveals uterine prolapse. Which stage of uterine prolapse is defined by the uterus located at the level of the hymen?

Stage 2 dx:Uterine prolapse Stage 1 is characterized by a pelvic organ that is > 1 cm above the hymen. Stage 2 is characterized by a pelvic organ extending from 1 cm above to 1 cm below the hymen. Stage 3 is characterized by a pelvic organ located > 1 cm past the hymen without complete uterine prolapse. Stage 4 is characterized by complete uterine prolapse.

A 72-year-old woman presents to the office with complaints of a new vaginal mass. While she was cleaning her house earlier in the day, she reports feeling a sudden fullness vaginally. She is worried her bladder is falling out. She is experiencing some discomfort that is worse when she is sitting. On physical exam, the external genitalia appear normal, and there is no frank blood. When the patient performs a Valsalva maneuver, the uterine cervix protrudes through the vaginal introitus by 5 mm. When relaxed, the prolapse is completely reduced. Bimanual exam reveals a 4 cm uterus with a smooth contour and no adnexal masses. There is anterior vaginal wall laxity. Which stage of pelvic organ prolapse is this patient experiencing?

Stage 3 stage 0 (no evidence of prolapse), stage 1 (prolapse further than 1 cm from the vaginal introitus), stage 2(prolapse ≤ 1 cm to the vaginal introitus), stage 3 (prolapse outside the vaginal introitus but by no more than 1 cm), and stage 4 (prolapse ≥ 1 cm outside the vaginal introitus). comparison Stage 1 is characterized by a pelvic organ that is > 1 cm above the hymen. Stage 2 is characterized by a pelvic organ extending from 1 cm above to 1 cm below the hymen. Stage 3 is characterized by a pelvic organ located > 1 cm past the hymen without complete uterine prolapse. Stage 4 is characterized by complete uterine prolapse. Risk factors for pelvic organ prolapse include advancing age, parity, obesity, menopause, and chronic constipation (which leads to straining). most common risk factor :Childbirth and pregnancy Which muscles comprise the levator ani muscle complex that provides primary support to the pelvic organs? Answer: Pubococcygeus, puborectalis, and iliococcygeus muscles. While inserted and in the correct place, the pessary reduces the prolapse and offers the patient relief from discomfort or emotional distress. Pessaries may also help reduce bowel and bladder symptoms if rectocele or cystocele is present. Pelvic floor rehab and pessary use are considered conservativetreatment measures.

A 32-year-old woman presents to the ED at 39 weeks of gestation in active labor. Her axillary temperature is 98.6°F, pulse is 88 beats per minute, and blood pressure is 120/80 mm Hg. Upon vaginal examination, an umbilical cord is noted to be protruding from the cervix which is dilated to five centimeters. While awaiting a surgical suite for an emergency cesarean section, which of the following agents would be most appropriate to administer?

Subcutaneous terbutaline dx:Umbilical cord prolapse Presenting signs of umbilical cord prolapse are often changes on fetal heart rate tracings, such as variable late decelerations or prolonged bradycardia. On vaginal exam, the umbilicus may be visible (overt umbilical cord prolapse) or not visualized (occult prolapse). Manual elevation of the fetal head with two fingers is the most common intervention employed to keep the umbilical cord free from compression. Placing the patient in Trendelenburg position or in a knee-to-chest position with her face towards the floor may change the relationship between the fetus and the umbilical cord and alleviate cord compression. Emergency cesarean section is the standard obstetrical management. If a delay in cesarean section is anticipated, such as waiting for an open surgical suite or transporting the patient from a remote location, the tocolytic terbutaline 0.25 mg can be given subcutaneously to decrease uterine contractions and alleviate pressure on the cord. True or false: Induction of labor is associated with an increased risk of umbilical cord prolapse. Answer: True.

A 40-year-old woman presents to the clinic for her annual Pap test. On bimanual exam, right adnexal fullness is noted. Transvaginal ultrasound reveals a pedunculated mass near the right ovary that is attached to the uterus by a stalk of tissue. On surgical removal, this mass is determined to be a uterine leiomyoma. Which of the following classifications of leiomyoma is it

Subserosal myoma Uterine Fibroids (Leiomyoma) Common during reproductive-ages Menorrhagia and dysmenorrhea PE will show a enlarged, asymmetric, and nontender uterus Diagnosis is made by pelvic ultrasound Majority do not require surgical or medical treatment Severe cases: myomectomy (fertility can be preserved) or hysterectomy

A 23-year-old woman returns to her obstetrician seven days postpartum. She is concerned with severe pain in the perineum and discharge following the sensation of "popping" in that area. She has been running a fever for the past day. A fourth-degree laceration was repaired following labor. Physical exam shows separated sutures and an erythematous perineal wound. What is the appropriate management of this patient?

Systemic antibiotics, debridement, and secondary repair At what time can sexual activity be resumed postpartum with an episiotomy? Answer: Sexual activity can be resumed postpartum after six weeks regardless of episiotomy.

A 26-year-old woman presents for routine prenatal care at 41 weeks gestation. You decide to induce labor. Which of the following is a known side effect of the medication typically used for labor induction?

Tachysystole

A 42-year-old woman presents to the office due to increasing intermittent breast pain and tenderness that has gotten increasingly worse over the past year. She states the pain peaks about 1 week before her period. She has tried wearing more supportive bras and taking over-the-counter pain relievers for the past 6 months but nothing has helped. She feels like she is not able to work when she experiences this pain. She notices her breasts feel heavy and extremely tender, especially with physical activity. Physical exam reveals very dense breasts upon palpation with rope-like texture bilaterally. Which of the following therapy is the best next step for the treatment of her condition?

Tamoxifen dx:Fibrocystic breast changes What are the side effects of tamoxifen? Answer: Hot flashes, vaginal dryness, joint pain, leg cramps. Tamoxifen also increases the risk of blood clots, stroke, uterine cancer, and cataracts. tx pathway Conservative treatment for a period of at least 6 months is the first-line treatment and includes wearing well-fitting and supportive bras, avoiding trauma to the breasts, applying heat to the breasts, and taking over-the-counter pain relievers to help alleviate symptoms. Women with more severe symptoms can be treated with second-line therapies such as tamoxifen or danazol. Tamoxifen is the preferred second-line treatment of choice since it has fewer side effects than danazol. Symptoms improve after menopause in women who are not receiving hormone replacement.

A 29-year-old G3P1102 pregnant woman presents to labor and delivery at 38 weeks and 0 days gestation complaining of leakage of fluid occurring 12 hours prior to arrival. Vital signs are T of 102.3°F, HR of 125 bpm, BP of 110/75 mm Hg, RR of 20/min, and oxygen saturation of 98% on room air. Physical examination reveals uterine tenderness and purulent drainage from the cervix with speculum exam. Laboratory studies show a white blood cell count of 14,000/μL and a C-reactive protein of 8.1 mg/L. Which of the following is one of the diagnostic criteria for the most likely diagnosis?

Temperature above 102.2°F dx:Intra-amniotic infection, also called chorioamnionitis, . Risk factors include prolonged length of labor and prelabor rupture of membranes. The symptoms and signs include fever, uterine tenderness, purulent or malodorous amniotic fluid, maternal tachycardia, and fetal tachycardia. Fever is an important finding because it is sensitive for the diagnosis. The presumptive diagnosis of intra-amniotic infection can be made in patients with a fever and at least one of the following: purulent-appearing fluid coming from the cervical os visualized during speculum examination, maternal white blood cell count > 15,000/μL, and a baseline fetal heart rate of at least 160 bpm for at least 10 minutes. The treatment of intra-amniotic infection consists of prompt augmentation or induction of labor and antibiotics. Intra-amniotic infection is not an indication for cesarean delivery, as the risk of wound infection and endometritis is increased if cesarean delivery is performed. The recommended antibiotic regimen consists of ampicillin and gentamicin. Patients who have an indication for a cesarean delivery should also be treated with metronidazole or clindamycin to provide coverage against anaerobes

A 30-year-old woman presents to the clinic complaining of infertility. She says she has had painful menses and pain with intercourse for years. Transvaginal ultrasound shows a left-sided ovarian cystic mass with homogeneous low-medium-level echoes. Which of the following exam findings supports the most likely diagnosis?

Tenderness in the posterior vaginal fornix dx: endometriosis The medical treatments for infertility due to endometriosis include ovulation-inducing medications, such as clomiphene, and intrauterine insemination. Some patients require assisted reproductive technology.

A 24-year-old G1P0 woman at 12 weeks gestation by last menstrual period presents to her obstetrician for her initial prenatal visit. A routine nucleic acid amplification test is positive for Chlamydia trachomatis. The patient is treated with a single dose of azithromycin. Which of the following is the best next step in the management of this patient?

Test of cure in 4 weeks

A 47-year-old woman presents to the clinic due to irregular menstrual cycles for the past 6 months. She reports that the time between her menstrual cycles has increased. She also reports hot flashes and difficulty sleeping. Which of the following is the best way to confirm the suspected cause of her symptoms?

The cause is best identified clinically dx:menopausal transition

A 22-year-old woman presents with a new lump in her right breast that she noticed while she was doing a self-exam in the shower about one week ago. The lump has not changed in size. Upon physical exam, you note a 2 cm discrete lump that is round, mobile and nontender to palpation to the upper outer quadrant of the right breast. The lump is solitary and you do not notice any other abnormalities. Which of the following statements is most likely given the suspected diagnosis?

They usually regress after menopause Dx: Fibroadenomas

A 26-year-old woman presents to the office due to worsening depression, mood swings, anxiety, low energy, and poor concentration that begins about 2 weeks before her period and lasts until her period starts. She states she has had these symptoms since she was a teenager, but lately they have gotten so severe she has had to take many days off work and is currently at risk of losing her job. A physical examination is unremarkable. Which of the following diagnostic tests should be ordered in the workup of her condition?

Thyroid-stimulating hormone test dx:Premenstrual dysphoric disorder (PMDD) is a severe form of premenstrual syndrome that is characterized by symptoms such as depression, anxiety, irritability, and anger during the last week of the luteal phase To diagnose PMDD, other underlying conditions that exhibit similar symptoms must be evaluated for, including thyroid disease, anemia, perimenopause, and menopause. Therefore, a thyroid-stimulating hormone test, human chorionic gonadotropin, complete blood count, and follicle-stimulating hormone level should be ordered.

A 25-year-old woman presents to the emergency department with a painful, swollen left breast and low-grade fever. She reports she developed mastitis while breastfeeding about 2 weeks ago that initially improved with antibiotics. She reports smoking one pack of cigarettes daily but always outside, away from the baby. She is taking no medications other than acetaminophen. She no longer has pain with breastfeeding but has developed a hot, painful, swollen area on her breast. On exam, she is obese but otherwise appears well. Her left breast has a localized, indurated area of about 3 cm diameter to the left of the areola, which is warm, tender to palpation, and fluctuant. Bedside ultrasound confirms the diagnosis and also facilitates aspiration of fluid. Which of the following elements of this patient's history puts her at greatest risk for this being a recurrent problem?

Tobacco use dx: breast abcess What type of breast cancer is associated with breast abscess? Answer: Inflammatory breast cancer.

A 25-year-old woman presents to her primary care physician complaining of nausea and breast tenderness. She is sexually active, last unprotected sexual intercourse was 9 days ago. She states she does not remember the date of her last menstruation and that her cycles are irregular. A home pregnancy test this morning was negative as is the in-office urine pregnancy test. Which of the following is the best next step in management?

Urine pregnancy test 14 days after last unprotected intercourse minimize the possibility of a false negative.

A 60-year-old woman presents to the clinic complaining of urinary incontinence. She says the symptoms primarily occur when she is laughing or sneezing. She has tried losing weight, reducing caffeine and alcohol intake, restricting water intake, and performing Kegel exercises without improvement. During pelvic examination, you notice vulvovaginal pallor, loss of vaginal rugae, and decreased elasticity. Which of the following is the best next step in management?*********

Topical estrogen dx stress incontinence Duloxetine (A) is a serotonin and norepinephrine reuptake inhibitor that has been studied for the treatment of stress urinary incontinence. However, it is predominantly used in patients who have stress urinary incontinence and another condition that is a more common indication for duloxetine, such as depression. Duloxetine is not approved for use by the U.S. Food and Drug Administration to treat stress urinary incontinence. Mirabegron (B) is a beta-agonist that can be used for the treatment of urge urinary incontinence. It is primarily used in patients who do not improve with antimuscarinic agents or who cannot tolerate antimuscarinic agents due to the adverse effects. Mirabegron is an incorrect answer because the patient has symptoms suggestive of stress urinary incontinence. Oxybutynin (C) is an antimuscarinic agent that can be used to treat urge urinary incontinence. These medications work by increasing bladder capacity and decreasing urinary urgency by blocking muscarinic receptors on the bladder from acetylcholine stimulation. Oxybutynin is an incorrect answer because the patient in the vignette has symptoms of stress urinary incontinence.

Which of the following tests would be performed first in a reverse-sequence screening algorithm for syphilis? What is the recommended next step in the reverse-sequence algorithm for syphilis if the treponemal test is positive and the reflex nontreponemal test is negative?

Treponemal enzyme immunoassays (EIA), FTA-abs,TPHA The traditional screening algorithm for syphilis begins with a nontreponemal test= VDR, RPR What is the recommended next step in the reverse-sequence algorithm for syphilis if the treponemal test is positive and the reflex nontreponemal test is negative? Answer: Perform a different treponemal test to confirm the initial test results. TX:Primary or secondary: IM benzathine penicillin G, 1 dose Tertiary: IM benzathine penicillin G qwk for 3 weeks

A 26-year-old woman presents to the emergency department with complaints of lower abdominal pain and vaginal discharge for the past 5 days. She also reports chills and malaise. Vital signs are T of 100.3°F, HR of 105 bpm, BP of 145/94 mm Hg, and RR of 20/min. On physical exam, she has tenderness to palpation in the left lower quadrant without guarding or rebound. Pelvic exam reveals purulent discharge from the cervical os, cervical motion tenderness, and left-sided adnexal tenderness. Transvaginal ultrasound shows an inflammatory multilocular mass in the left adnexal area. Which of the following is the most likely diagnosis?

Tubo-ovarian abscess without rupture

A 26-year-old woman presents to the emergency department with complaints of lower abdominal pain and vaginal discharge for the past 5 days. She also reports chills and malaise. Vital signs are T of 100.3°F, HR of 105 bpm, BP of 145/94 mm Hg, and RR of 20/min. On physical exam, she has tenderness to palpation in the left lower quadrant without guarding or rebound. Pelvic exam reveals purulent discharge from the cervical os, cervical motion tenderness, and left-sided adnexal tenderness. Transvaginal ultrasound shows an inflammatory multilocular mass in the left adnexal area. Which of the following is the most likely diagnosis?

Tubo-ovarian abscess without rupture What is the most common age group to develop a tubo-ovarian abscess? Answer: 15-25 years of age. Treatment of a TOA involves antibiotics and sometimes drainage. The diagnosis is further supported by surgical findings if drainage is performed. Patients diagnosed with a TOA should be admitted for intravenous antibiotics. There are multiple appropriate antibiotic regimens, but cefoxitin and doxycycline or cefotetan and doxycycline are common choices. A patient with TOA meeting the following criteria should be treated initially with antibiotics alone: hemodynamically stable, no signs of an acute abdomen, abscess size < 7 cm in diameter, and premenopausal status. Furthermore, patients with an abscess > 7 cm who meet the rest of the above criteria may also undergo a trial of antibiotics alone. Those who are not improving or are worsening with antibiotics alone often require further intervention. Some patients can be treated with minimally invasive abscess drainage, while others require surgical treatment. Postmenopausal women with a suspected TOA are at higher risk for malignancy, thus surgical intervention is recommended initially for diagnostic and therapeutic purposes.

A 48-year-old premenopausal woman is being evaluated for an adnexal mass palpated on bimanual exam. A pelvic ultrasound reveals a fixed right ovarian 6 cm complex mass and a large amount of free fluid. CA-125 is 193 units/mL, (carcinoembryonic antigen) CEA is 2.8 ng/mL. Which of the following supports referral to a gynecologic oncologist?

Ultrasound findings dx ovarian cancer premenopausal women include ultrasound findings suggestive of malignancy (free fluid consistent with ascites, or a nodular fixed mass), or evidence of distant metastases. In this case, the ultrasound finding should dictate referral to an oncologist.

A 37-year-old woman presents to the office to ask about female contraceptive barrier methods and wants to be fitted for a cervical cap or diaphragm. Which of the following is a side effect of using these devices?

Urinary tract infection

A 27-year-old G3P2002 presents to the labor and delivery unit at 32 6/7 weeks gestational age with lower abdominal cramping. She reports the cramping pain has been increasing in frequency throughout the day but reports no vaginal bleeding or leakage of fluids. A fetal fibronectin test is performed and is negative. Which of the following findings would meet the diagnostic criteria for preterm labor?

Uterine contractions 5 minutes apart and a cervical dilation of 3 cm prior to 37 weeks of gestational age. The specific measurements of cervical change that meet the criteria for preterm labor include at least one of the following: cervical dilation ≥ 3 cm, cervical length < 20 mm on transvaginal ultrasound, or cervical length between 20 and 30 mm with a positive fetal fibronectin. -------------------- tx no signs of obstetric complications, such as placental abruption, prelabor rupture of membranes, or intra-amniotic infection. Women who are at ≥ 34 weeks gestation should not be given antenatal corticosteroids or tocolytic (anticontraction) agents. Women who are < 34 weeks of gestation presenting with preterm labor should be treated with tocolytic therapy for 48 hours, antibiotics for group B streptococcal prophylaxis, and antenatal corticosteroids for 48 hours. Magnesium sulfate is also recommended for women in preterm labor who are between 24 and 32 weeks gestational age. The goal of treatment with tocolytic agents and antibiotics is to delay labor for 48 hours, which allows time for the corticosteroids to improve fetal lung development and the patient to be transferred (if necessary) to the best facility for premature infants. Indomethacin is the tocolytic agent typically used for women with preterm labor who are between 24 and 32 weeks gestational age. Women who are between 32 and 34 weeks gestation who are candidates for tocolysis are treated with nifedipine. Antenatal corticosteroids, such as betamethasone or dexamethasone, are recommended for women with preterm labor who are between 22 weeks and 33 6/7 weeks gestational age. Women in preterm labor should be given antibiotic prophylaxis against group B streptococcal infection. Women in preterm labor who are between 24 and 32 weeks gestational age are given magnesium sulfate while in labor for neuroprotection. The following interventions are ineffective in preventing preterm birth in women who have acute preterm labor: bedrest, hydration, sedatives, antibiotics, and progesterone supplementation.

A 25-year-old woman is recovering from uncomplicated vaginal delivery she had 24 hours ago. Which of the following physiologic changes is a major mechanism preventing postpartum hemorrhage?

Uterine involution

A 52-year-old woman presents to the gynecology clinic with vaginal dryness. She also reports vaginal burning, dyspareunia, and urinary frequency. The patient reports no hot flashes. Pelvic examination reveals vulvovaginal erythema and fissuring. Which of the following is the initial first-line treatment for the most likely diagnosis?

Vaginal moisturizers and lubricants dxVulvovaginal (urogenital) atrophy ------------------------------------------- Vaginal estrogen (C) is recommended as the initial hormonal treatment for vulvovaginal atrophy. However, vaginal moisturizers and lubricants are recommended as the initial treatment due to their safety and possible efficacy.

A 60-year-old woman presents to her gynecologist with complaints of urinary frequency, urgency and incontinence. A cystocele is diagnosed on pelvic exam. The patient has undergone pelvic floor muscle training but symptoms have only improved slightly. What is the mainstay of nonsurgical treatment for this patient?

Vaginal pessary dx:cystocele mainstay of nonsurgical treatment in women with pelvic organ prolapse, in this case cystocele. What term is used to describe the condition of all three of the compartments herniating through the vaginal introitus? Answer: Uterine procidentia. -------------------------------------------------------- Mirabegron (B) is a β3 adrenergic receptor agonist used in the treatment of overactive bladder. Oxybutynin (C) is an antimuscarinic medication that is also used in the treatment of overactive bladder.

Which of the following vitamins, if taken in excess, is teratogenic in the first trimester and is associated with spontaneous abortion and fetal malformation?

Vitamin A

A 44-year-old woman with uncontrolled diabetes mellitus presents to the gynecologic clinic with vulvar pruritus, vaginal burning, vaginal irritation, and dysuria. Physical examination reveals vulvar erythema. Urinalysis has no leukocyte esterase or nitrites. The vaginal pH is 4.25. Which of the following is the most common finding in the most likely condition?

Vulvar pruritus dx:Vulvovaginal candidiasis What is the recommended treatment for vulvovaginal candidiasis in pregnant women? Answer: Topical miconazole or clotrimazole applied vaginally for 7 days. Patients with uncomplicated infection are treated with oral fluconazoleadministered as a single oral dose. Patients with vulvovaginal candidiasis and uncontrolled diabetes mellitus are considered to have complicated infections. Patients with complicated infection are treated with two doses of oral fluconazole administered as oral doses 3 days apart.

A 41-year-old nulliparous woman with a history of infertility presents to her gynecologist with nausea, vomiting, and pelvic discomfort. Her last menstrual period was 7 weeks ago. A serum human chorionic gonadotropin level is obtained and is 346,000 mIU/mL. A transvaginal ultrasound reveals the absence of amniotic fluid and a snowstorm pattern (like a bag of graps). The patient is scheduled for surgical treatment. Which of the following is the most appropriate follow-up for the patient after surgery?

Weekly monitoring of serum human chorionic gonadotropin until < 5 mIU/mL dx Hydatidiform mole

A 41-year-old nulliparous woman with a history of infertility presents to her gynecologist with nausea, vomiting, and pelvic discomfort. Her last menstrual period was 7 weeks ago. A serum human chorionic gonadotropin level is obtained and is 346,000 mIU/mL. A transvaginal ultrasound reveals the absence of amniotic fluid and a snowstorm pattern. The patient is scheduled for surgical treatment. Which of the following is the most appropriate follow-up for the patient after surgery?

Weekly monitoring of serum human chorionic gonadotropin until < 5 mIU/mL dx:Hydatidiform mole, also known as a molar pregnancy, What is the most common site of early hematogenous metastases of a choriocarcinoma? Answer: Lungs.

24-year-old woman presents to the office for evaluation of infertility after trying to conceive for several years. A transabdominal ultrasound reveals a "string of pearls" appearance to the bilateral ovaries. Which of the following clinical findings will she most likely have on a history and physical examination?

Weight gain, hirsutism, acne, and insulin resistance dx:Polycystic ovarian syndrome (PCOS) Rotterdam criteria require that, in order to make the diagnosis of PCOS, there must be at least two out of three of the following: oligo- or anovulation or both, evidence of hyperandrogenism, and the presence of polycystic ovaries on ultrasound. exercise and weight reduction, which can decrease androgen and insulin levels. The mainstay of treatment is oral contraceptives. Other pharmacological agents include clomiphene for anovulation, diabetic agents (e.g., metformin, which helps with both insulin resistance and can induce ovulation), antiandrogens, and statins.

A 25-year-old G1P0 at 39 weeks' gestation has been admitted to the labor and delivery department. On admission, a digital cervical examination was performed. Which of the following best describes the appropriate time for repeat cervical examinations during labor? What is the purpose of repeat digital cervical examinations during labor? Answer: To monitor the progression of dilation and determine effacement and fetal station.

When the woman feels the urge to push The first cervical examination should be performed at the time of admission. During the first stage of labor, the examinations should be performed every two to four hours. During stage two, every one to two hours is sufficient. When the woman feels the urge to push, a cervical examination is done to determine whether the cervix is completely dilated. If the woman has chosen to have anesthesia, a cervical examination should be done prior to its administration. Additionally, if the fetal heart rate becomes irregular, a cervical exam is indicated to evaluate for complications such as cord prolapse or uterine rupture. The results of the examination can be documented on a partogram, a graphical representation of the progression of dilation, in addition to the medical record.

Which of the following would increase the chance of intrauterine device expulsion or failure?

ages 25 years or less. Increased in postpartum insertion or insertion following a second trimester abortion. Insertion of an IUD while breastfeeding increases the possibility of uterine perforation. The contraindications to IUD use include uterine anomalies and leiomyoma that distort the uterine cavity. IUDs are generally safe and effective, however, there are complications that should be explained to the patient at the time of insertion. There is a risk of pelvic inflammatory disease, expulsion of the device, failure of contraception with increased risk of ectopic pregnancy, perforation, or malposition of the device. The most common side effects that may result in the removal of the device are pain and bleeding.

At which of the following ages does the United States Preventive Services Task Force recommend initiating screening for the suspected diagnosis of Breast cancer?

at 50 years of age and screening every 2 years with a mammogram in average-risk women.

Patients with Lynch syndrome are also at an increased risk for

both endometrial and colon cancers.

trichomoniasis complication in preganacy and follow up on tx management

complication : risk of HIV transmission and complication followup with NAAT 3 months after TX flagl 2g oral once or 500mg BID 7 days

A 24-year-old woman presents with a headache, fever, and an erythematous rash on the flanks that she noticed yesterday. She reports that she had a painless vaginal ulcer that resolved 5 weeks ago. On exam, you note a maculopapular rash on the trunk as well as on the palms and soles. Which of the following additional physical findings is most likely to be present?

condyloma latum (wart-like lesions on the genitals) is seen in patients with secondary syphilis. Patients with secondary syphilis may also have a flu-like illness (headache, sore throat, malaise, or fever), aseptic meningitis, hepatitis, or alopecia. dx syphillis secondary Secondary syphilis develops over 4 to 8 weeks after the initial painless chancre has resolved. Patients with secondary syphilis classically present with a maculopapular rash affecting the trunk and extremities and often involve the palms and soles. Additionally,...................... The diagnosis of syphilis is made by dark-microscopy (gold standard), which shows the visualization of spirochetes, although this is seldom performed clinically. Serologic tests include nontreponemal tests (rapid plasma reagin, venereal disease research lab), and treponemal tests (fluorescent treponemal antibody absorption, microhemagglutination assay for Treponema pallidum). Nontreponemal tests are ideal for screening, and if positive, should be confirmed using specific treponemal tests. Benzathine penicillin G that is given intramuscularly is the preferred agent for syphilis. What is an alternative antibiotic medication that can be used to treat syphilis in the setting of penicillin hypersensitivity? Answer: Tetracycline or doxycycline.

A 53-year-old woman presents to the office complaining of hot flashes. She states that they interfere with her daily activities as well as with her sleep at night. Her past medical and surgical history is significant for asthma and a vaginal hysterectomy for abnormal uterine bleeding. Which of the following is the best option for this patient to provide the most relief?

estrogen dx postmenopausal True or false: black cohosh is an effective treatment for hot flashes. Answer: False. While it is widely used there is not sufficient evidence to support its effectiveness.

Condylomata acuminata

gential bumps that are pruritic and not painful. Physical exam reveals multiple papillomatous skin-colored lesions, as seen on the image above, on the labia and perianal area. Which of the following is the most likely diagnosis?

The risk factors for shoulder dystocia

include weight over 4,000 g, maternal diabetes mellitus, previous shoulder dystocia, excess maternal weight gain during pregnancy, post-term pregnancy, and male fetus.

A 60-year-old woman presents to the clinic reporting a rapidly enlarging, tender mass in her left breast. She was treated with antibiotics 3 weeks ago when she first noticed the mass, but her symptoms did not abate. She is afebrile, and other vital signs are within normal limits. On exam, an area of erythema involving the entire inferior half of the left breast is noted. The area is tender to palpation, with a firm mass underlying the erythema, and the skin of the breast is edematous, causing the pores of the skin to be very pronounced. There is a palpable, immobile, nontender left axillary node as well. Which of the following is the most likely diagnosis?

inflammatory breast cancer #peau d orange Inflammatory Breast Cancer Patient presents with edema and erythema that mimics mastitis Skin resembles an orange peel (peau d'orange) due to tumor emboli blocking lymphatic tissue Rare, high-grade, rapidly progressive, poor prognosis Tx: neoadjuvant chemotherapy → mastectomy with node dissection → chest wall radiation

Which of the following best represents a positive prognostic variable for successful external cephalic version of a breech fetus? name the risk of doing external cephalic version?

nonlongitudinal fetal lie, and whose fetus has an unengaged presenting part.Nonlongitudinal lie aids in external cephalic version because the fetus need not be rotated 180 degrees Risks of external cephalic version include stillbirth, placental abruption, emergency cesarean section, cord prolapse, vaginal bleeding, rupture of membranes, maternofetal transfusion, and transient abnormal changes in fetal heart rate. The rate of serious complications in large studies was found to be very low.

1)Why is the right-sided ovary more likely to torse than the left? Answer: Because the right utero-ovarian ligament is longer than the left and the sigmoid colon is on the left side, so the left ovary has less ability to move and twist. 2)Which organisms most often cause bacteremia secondary to intra-amniotic infection? Answer: Escherichia coli and group B Streptococcus. 3)What is Piskacek sign in pregnancy? Answer: Asymmetrical enlargement of the uterus due to lateral implantation. 4)What are the risk factors for low Apgar scores? Answer: Premature birth, cesarean delivery, and perinatal complications. 5)In which phase of the menstrual cycle does cyclic breast pain occur? Answer: Luteal phase. 6)Which organism causes lymphogranuloma venereum? Answer: Chlamydia trachomatis. 7)At which level of cervical dilation is a patient considered to be in active labor? Answer: ≥ 6 cm. 8)In shoulder dystocia, which maneuver is used initially to release an impacted shoulder by sharply flexing the mother's legs so that the thighs touch the abdomen? Answer: McRoberts maneuver. 9)What skin change is associated with lymphatic obstruction of the breast? Answer: Peau d'orange, which is associated with progressive breast cancer. 10)Why do hyperthyroidism and theca lutein cysts occur during gestational trophoblastic disease? Answer: The alpha subunit of human chorionic gonadotropin closely mimics thyroid-stimulating hormone and luteinizing hormone. 11)What is the recommended route, dosage, and duration of metronidazole prescribed for pregnant patients with bacterial vaginosis? Answer: Metronidazole 500 mg PO bid for 7 days. 12)What is the presenting part of the fetal head in a position of asynclitism? Answer: The parietal bone. 13)What are the adverse effects of gonadotropin-releasing hormone agonists? Answer: Hot flashes and decreased bone mineral density. 14)What value does a follicle-stimulating hormone level need to be consistently elevated above to be indicative of menopause? Answer: 30 mIU/mL. 15)What is the name for the type of prolapse that involves the herniation of the anterior, posterior, and apical compartments simultaneously? Answer: Procidentia. 16)What vaginal pH is associated with vaginal atrophy? Answer: ≥ 5. 17)In which category of the Breast Imaging Reporting and Data System (BI-RADS) do most breast cysts belong? Answer: BI-RADS 2. 18)Which karyotype is seen in patients with androgen insensitivity syndrome? Answer: 46,XY. 19)What are the common side effects of selective serotonin reuptake inhibitors? Answer: Decreased libido, delayed orgasm, nausea, weight gain, headache, and insomnia. 20)How are recurrent abortions defined? Answer: ≥ 2 abortions that occur in sequence in previously healthy pregancies that are < 24 weeks gestation. 21)When should women with hypertensive disorders follow-up for a clinic visit following delivery? Answer: Within 7 days. 21)Women with the BRCA1 gene mutation are more likely to be diagnosed with what form of breast cancer? Answer: Medullary carcinoma. 22)Which breech presentation is the most common? Answer: Frank breech. 23)What finding on thoracentesis of a patient with a pleural effusion suggests a diagnosis of epithelial ovarian cancer? Answer: Malignant müllerian cells. 24)Which muscles comprise the levator ani muscle complex that provides primary support to the pelvic organs? Answer: Pubococcygeus, puborectalis, and iliococcygeus muscles. 25)Which drug is considered the first-line corticosteroid during preterm labor? Answer: Betamethasone. 26)What is the preferred treatment for syphilis? Answer: Penicillin G benzathine 2.4 million units IM. 27)When can magnesium sulfate be discontinued? Answer: At least 24 hours after delivery. 28)What is the average length of an umbilical cord? Answer: 50 cm. 29)What are the greatest risk factors for developing placenta previa? Answer: Multiparity, smoking, and increasing maternal age. 30)Which ligaments support the uterus and attach the cervix to the posterior surface of the pubic symphysis? Answer: Pubocervical ligaments.

q& A

1)What is a more severe form of premenstrual syndrome? Answer: Premenstrual dysphoric disorder. 2)Which complication of loop electrosurgical excision procedure is characterized by recurrent second-trimester miscarriage? Answer: Cervical insufficiency. 3)What is a serious complication of venous compression due to large uterine leiomyomatous disease? Answer: Venous thromboembolism. 4)What is the name of the medical maneuver used to resolve shoulder dystocia, where the maternal thighs are flexed against the maternal abdomen to reduce the sacral-spinal angle? Answer: McRoberts technique. 5)What physiological role does follicle-stimulating hormone play in the human body? Answer: Follicle-stimulating hormone is a gonadotropin that stimulates the growth of ovarian follicles in women and helps the maturation of sperm in men. 6)True or false: patients with amenorrhea should always have a pregnancy test. Answer: True. 7)What are the causes of fetal tachycardia? Answer: Fetal movement, maternal factors (anemia, hyperthyroidism, hypoxia, dehydration, fever or sepsis, and anxiety), fetal factors (dysrhythmias), and placental abruption. 8)What is the recommended prophylactic antibiotic in pregnant women with a history of extended-spectrum beta-lactamase-producing Enterobacteriaceae? Answer: Meropenem. 9)How long after menarche do ovulatory cycles typically start? Answer: 2-5 years. 10)hat are the risk factors for pelvic organ prolapse? Answer: Increased parity, advancing age, obesity, and increased intra-abdominal pressure. 11)Bitemporal vision loss is a sign suggesting what etiology of abnormal uterine bleeding? Answer: Prolactinoma. 12)True or false: breastfeeding is currently recommended for infants born to women with confirmed Zika virus infection. Answer: True. 13)True or false: the copper intrauterine device is contraindicated during breastfeeding. Answer: False. 14)What are the common causes for acetowhite changes seen with colposcopy? Answer: Inflammation, subclinical papillomavirus infection, cervical intraepithelial neoplasia, and metaplasia 15)What type of twins occurs with division of a fertilized egg 13 days or later after fertilization? conjoined twins 16)Which antibiotic should be prescribed for periductal mastitis? Answer: Amoxicillin-Clavulanate. 17)What are some side effects of magnesium sulfate? Answer: Depressed reflexes, hypotension, flushing, drowsiness, impaired cardiac function, diaphoresis, and vision changes. 18)What is the name for the lacy, white plaques seen on the buccal mucosa of patients with lichen planus? Answer: Wickham striae

q& A part 2

1)At how many weeks of pregnancy can screening with chorionic villus sampling be performed? Answer: 10-13 weeks.' 2)What is the recommended treatment for vulvovaginal candidiasis in pregnant women? Answer: Topical miconazole or clotrimazole applied vaginally for 7 days. 3)What is considered an acceptable decrease in human chorionic gonadotropin levels after treatment for gestational trophoblastic disease? Answer: A decrease > 10% demonstrated by four values taken weekly for 3 consecutive weeks. 4)Which medication can be started during the second trimester in high-risk patients to reduce the risk of developing preeclampsia? Answer: Low-dose aspirin. 5)What are the adverse effects of selective serotonin reuptake inhibitors? Answer: Sexual dysfunction, weight gain, weight loss, insomnia, drowsiness, and orthostatic hypotension. 6)What range of endometrial thickness is associated with a low risk of endometrial disease in postmenopausal women according to the American College of Obstetricians and Gynecologists? Answer: ≤ 4 mm. 7) What is the earliest gestation age an intrauterine pregnancy may be detected by transvaginal ultrasound? Answer: 5.5-6 weeks. Vulvar Cancer History of human papillomavirus (types 16, 18, 33) Vulvar lesion and pruritus PE will show unifocal vulvar ulcer, plaque, or mass, predominantly on the labia majora Most common type is squamous cell carcinoma (SCC) 8)Which of the following contraceptive options has a black box warning due to the risk of blood clots? Ethinyl estradiol and norelgestromin patch 9)True or false: the risk of venous thromboembolism is higher in women on combined hormonal contraception compared to pregnant women. Answer: False. 10)True or false: pregnant women with a history of genital herpes and prodromal symptoms but no active lesions at the time of labor should deliver vaginally. Answer: False. Prodromal symptoms are an indication for cesarean delivery. 11)What are possible long-term complications of pelvic inflammatory disease? Answer: Chronic pelvic pain, infertility, and ectopic pregnancy. 12)What are clue cells? Answer: Epithelial cells coated in bacteria, most commonly found in bacterial vaginosis. 13)True or false: the annual influenza vaccine is contraindicated in pregnancy. Answer: False. It is highly recommended because pregnant women can have more severe symptoms of influenza. 14)The ductus arteriosus connects which two fetal structures? Answer: The pulmonary artery and the descending aorta.

q&a

1)What is a heterotopic pregnancy? Answer: Pregnancy with one intrauterine gestational sac and one ectopic gestational sac. 2)What form of cervical cancer is linked closely to diethylstilbestrol exposure? Answer: Clear cell adenocarcinoma. 3)How does the normal vaginal pH compare to the vaginal pH in postmenopausal women? Answer: Postmenopausal pH is more basic. The normal pH in reproductive-aged women is 3.5-5.0, and the pH in postmenopausal women is often 5.5-6.8. 4)What are some commonly used tocolytic drugs? Answer: Magnesium sulfate, indomethacin, terbutaline, and nifedipine. 5)At which gestational age in the pregnancy are cervical cerclages typically removed? Answer: 36-37 weeks gestation or with the onset of preterm labor. 6)Excessive intake of which micronutrient during pregnancy is associated with fetal goiter? Answer: Iodine. 7)What type of cancer does administration of estrogen-only therapy in a perimenopausal woman with an intact uterus increase the risk of? Answer: Endometrial cancer. 8)Which artery does the uterine artery originate from? Answer: The anterior division of the internal iliac artery. 9)Which study evaluates for patency of the fallopian tube system? Answer: Hysterosalpingogram. 10)What is the medical term for the probability of achieving a pregnancy within one menstrual cycle? Answer: Fecundability 11)Which condition in pregnancy is characterized by an accumulation of blood between the uterine wall and the chorionic membrane? Answer: Subchorionic hematoma. 12) What are the four Ts (mnemonic) for causes of postpartum hemorrhage? Answer: Tone, trauma, tissue, and thrombin. 13)When is an infant considered full term? Answer: 39 weeks. 14) What other factors may cause a thickened endometrial stripe? Answer: The presence of endometrial polyps, obesity, diabetes, and current tamoxifen therapy. 15)At which vertebral level is an epidural most commonly placed in obstetrics patients? Answer: L3-L4. 16)Which condition refers to a fallopian tube that is blocked with fluid? Answer: Hydrosalpinx. 17)What defines a reactive nonstress test? Answer: At least two accelerations in a 20 minute period. 18)What is Mayer-Rokitansky-Hauser syndrome? Answer: It is the failure of the uterus and vagina to develop in women who have normal functioning ovaries and normal external genitalia. 19)When does ovulation resume after childbirth on average? Answer: After 45 days in nonlactating women and 189 days in lactating women. 20)True or false: sexual partners of women with bacterial vaginosis should also be treated. Answer: False. 21)What are some of the teratogenic effects of angiotensin-converting enzyme inhibitors? Answer: Fetal kidney hypoperfusion, fetal hypotension, fetal growth restriction, and fetal demise. 22)True or false: cigarette smoking is associated with an increased risk of squamous cell carcinoma of the cervix but not of adenocarcinoma. Answer: True. 23)Which of the following medications is recommended as the first-line treatment for this patient? PMDD Sertraline 24)Which type of surgery is considered a last resort in patients with severe, disabling symptoms from premenstrual dysphoric disorder? Answer: Bilateral salpingo-oophorectomy. 25)According to the U.S. Preventive Services Task Force, at what age is it recommended to start screening for breast cancer for those at average risk? Answer: 50 years. 26)A patient with a history of LSIL and a negative HPV test that has negative cytology and negative HPV at one year should have repeat cotesting at what time? Answer: 3 years later. 27)What condition is characterized by recurrent episodes of meningitis caused by herpes simplex virus type 2 (HSV-2)? Answer: Mollaret meningitis. Genital Herpes Simplex Patient presents with a painful genital rash PE will show grouped erythematous shallow cluster of vesicles and lymphadenopathy Labs will show multinucleated giant cells on Tzanck smear (poor sensitivity) Diagnosis is made clinically, gold standard is tissue culture with polymerase chain reaction (PCR) Most commonly caused by herpes simplex virus (HSV) type 2 Treatment is acyclovir Pregnancy: acyclovir for 7 days after primary infection and from 36 weeks to delivery 28)Which medication used to treat cystitis can be administered as a single dose? Answer: Fosfomycin.

q/a part 3

55-year-old woman presents to her primary care physician with a complaint of incontinence. She states that it is worse when she sneezes or when she lifts anything heavy. What is the best initial treatment to help alleviate these symptoms?

stress incontinence is Kegel exercises and lifestyle modificationsincluding weight loss and smoking cessation. If a six-week trial fails, more aggressive treatments may be considered. Bladder training and timed voiding may help to prevent leakage. Mid-urethral sling procedure is a surgical procedure that can be helpful in women who have not had sufficient success through non-invasive means.

A 54-year-old premenopausal woman is diagnosed with metastatic hormone receptor-positive, human epidermal growth factor receptor 2-negative breast cancer, for which she is prescribed a single-agent therapeutic treatment. Which of the following is considered a black box warning associated with this medication?

tamoxifen result in endometrial hyperplasia and uterine malignancy, which is a black box warning for this agent.

31-year-old G1P1 woman at 39 weeks gestation has a normal spontaneous vaginal delivery. The patient is evaluated immediately after delivering for possible obstetric perineal laceration. On physical examination, the patient is found to have a laceration through the perineal fascia and musculature extending into the external anal sphincter. Based on the description of the laceration, which of the following would be the patient's correct diagnosis?

third degree laceration


Ensembles d'études connexes

Chapter 4 Choosing A Form of Business Ownership

View Set

HEALTH ASSESSMENT MIDTERM MOCK REVIEW/TEST

View Set

(UNIT 3 - FUNCTIONS) Algebra 1 exam prep

View Set